Эдс индукции в чем измеряется: В чем измеряется ЭДС индукции?

Содержание

Эдс индукции

Причиной электродвижущей силы может стать изменение магнитного поля в окружающем пространстве. Это явление называетсяэлектромагнитной индукцией. Величина ЭДС индукции в контуре определяется выражением

где — поток магнитного поля через замкнутую поверхность , ограниченную контуром. Знак «−» перед выражением показывает, что индукционный ток, созданный ЭДС индукции, препятствует изменению магнитного потока в контуре (см. правило Ленца).

41. Индуктивность, ее единица СИ. Индуктивность длинного соленоида.

Индукти́вность (или коэффициент самоиндукции) — коэффициент пропорциональности между электрическим током, текущим в каком-либо замкнутом контуре, и магнитным потоком, создаваемым этим током через поверхность[1], краем которой является этот контур.[2][3][4].

В формуле

— магнитный поток, — ток в контуре, — индуктивность.

  • Нередко говорят об индуктивности прямого длинного провода(

    см. ). В этом случае и других (особенно – в не отвечающих квазистационарному приближению) случаях, когда замкнутый контур непросто адекватно и однозначно указать, приведенное выше определение требует особых уточнений; отчасти полезным для этого оказывается подход (упоминаемый ниже), связывающий индуктивность с энергией магнитного поля.

Через индуктивность выражается ЭДС самоиндукции в контуре, возникающая при изменении в нём тока[4]:

.

Из этой формулы следует, что индуктивность численно равна ЭДС самоиндукции, возникающей в контуре при изменении силы тока на 1 А за 1 с.

При заданной силе тока индуктивность определяет 

энергию магнитного поля, создаваемого этим током[4]:

.

В системе единиц СИ индуктивность измеряется в генри[7], сокращенно Гн, в системе СГС — в сантиметрах (1 Гн = 109см)[4]. Контур обладает индуктивностью в один генри, если при изменении тока на один ампер в секунду на выводах контура будет возникать напряжение в один вольт. Реальный, не сверхпроводящий, контур обладает омическим сопротивлением R, поэтому на нём будет дополнительно возникать напряжение U=I*R, где I — сила тока, протекающего по контуру в данное мгновение времени.

Символ , используемый для обозначения индуктивности, был взят в честь Ленца Эмилия Христиановича (Heinrich Friedrich Emil Lenz)[источник не указан 1017 дней]. Единица измерения индуктивности названа в честь Джозефа Генри (Joseph Henry)[8]. Сам термин индуктивность был предложен Оливером Хевисайдом (Oliver Heaviside) в феврале 1886 года[источник не указан 1017 дней].

Электрический ток, который течет в замкнутом контуре, создает вокруг себя магнитное поле, индукция которого, согласно закону Био-Савара-Лапласа, пропорциональна току. Сцепленный с контуром магнитный поток Ф поэтому прямо пропорционален току I в контуре:  (1)  где коэффициент пропорциональности L называется

индуктивностью контура.   При изменении в контуре силы тока будет также изменяться и сцепленный с ним магнитный поток; значит, в контуре будет индуцироваться э.д.с. Возникновение э.д.с. индукции в проводящем контуре при изменении в нем силы тока называетсясамоиндукцией.  Из выражения (1) задается единица индуктивности генри (Гн): 1 Гн — индуктивность контура, магнитный поток самоиндукции которого при токе в 1 А равен 1 Вб: 1 Гн = 1 Вб/с = 1 В

·c/А . 

Вычислим индуктивность бесконечно длинного соленоида. Полный магнитный поток сквозь соленоид (потокосцепление) равен μ0μ(N2I/l)S . Подставив в (1), найдем  (2)  т. е. индуктивность соленоида зависит от длиныl солениода, числа его витков N, его , площади S и магнитной проницаемости μ вещества, из которого изготовлен сердечник соленоида.  Доказано, что индуктивность контура зависит в общем случае только от геометрической формы контура, его размеров и магнитной проницаемости среды, в которой он расположен, и можно провести аналог индуктивности контура с электрической емкостью уединенного проводника, которая также зависит только от формы проводника, его размеров и диэлектрической проницаемости среды.

  Найдем, применяя к явлению самоиндукции закон Фарадея, что э.д.с. самоиндукции равна  Если контур не претерпевает деформаций и магнитная проницаемость среды остается неизменной (в дальнейшем будет показано, что последнее условие выполняется не всегда), то L = const и(3)  где знак минус, определяемый правилом Ленца, говорит о том, чтоналичие индуктивности в контуре приводит к замедлению изменения тока в нем.  Если ток со временем увеличивается, то (dI/dt<0) и ξs>0 т. е. ток самоиндукции направлен навстречу току, обусловленному внешним источником, и замедляет его увеличение. Если ток со временем уменьшается, то (dI/dt>0) и ξ
s
<0 т. е. индукционный ток имеет такое же направление, как и уменьшающийся ток в контуре, и замедляет его уменьшение. Значит, контур, обладая определенной индуктивностью, имеет электрическую инертность, заключающуюся в том, что любое изменение тока уменьшается тем сильнее, чем больше индуктивность контура.

 

42. Ток при размыкании и замыкании цепи.

При всяком изменении силы тока в проводящем контуре возникает э. д. с. самоиндукции, в результате чего в контуре появляются дополнительные токи, называемые 

экстратоками самоиндукции. Экстратоки самоиндукции, согласно правилу Ленца, всегда направлены так, чтобы препятствовать изменениям тока в цепи, т. е. направлены противоположно току, создаваемому источником. При выключении источника тока экстратоки имеют такое же направление, что и ослабевающий ток. Следовательно, наличие индуктивности в цепи приводит к замедлению исчезновения или установления тока в цепи.

Рассмотрим процесс выключения тока в цепи, содержащей источник тока с э.д.с. , резистор сопротивлением R и катушку индуктивностью L. Под действием внешней э. д. с. в цепи течет постоянный ток

(внутренним сопротивлением источника тока пренебрегаем).

В момент времени t=0 отключим источник тока. Ток в катушке индуктивностью L начнет уменьшаться, что приведет к возникновению э. д.с. самоиндукции препятствующей, согласно правилу Ленца, уменьшению тока. В каждый момент време­ни ток в цепи определяется законом Ома I=s/R, или

(127.1)

Разделив в выражении (127.1) переменные, получим Интегрируя это уравнение по I (от I0 до I) и t (от 0 до 

t), находим ln (I /I0) = Rt/L, или

(127.2)

где =L/R  постоянная, называемая временем релаксации. Из (127.2) следует, что  есть время, в течение которого сила тока уменьшается в е раз.

Таким образом, в процессе отключения источника тока сила тока убывает по экспоненциальному закону (127.2) и определяется кривой 1 на рис. 183. Чем больше индуктивность цепи и меньше ее сопротивление, тем больше  и, следовательно, тем медленнее уменьшается ток в цепи при ее размыкании.

При замыкании цепи помимо внешней э. д. с.  возникает э. д. с. самоиндукции препятствующая, согласно правилу Ленца, возрастанию тока. По закону Ома, или

Введя новую переменную преобразуем это уравнение к виду

где  — время релаксации.

В момент замыкания (t=0) сила тока I = 0 и u = –. Следовательно, интегрируя по и (от – до IR) и t (от 0 до t), находим ln[(IR)]/–= t/, или

(127.3)

где — установившийся ток (при t).

Таким образом, в процессе включения источника тока нарастание силы тока в цепи задается функцией (127.3) и определяется кривой 2 на рис. 183. Сила тока возрастает от начального значения 

I=0 и асимптотически стремится к установившемуся значению . Скорость нарастания тока определяется тем же временем релаксации =L/R, что и убывание тока. Установление тока происходит тем быстрее, чем меньше индук­тивность цепи и больше ее сопротивление.

Оценим значение э.д.с. самоиндукции , возникающей при мгновенном увеличении сопротивления цепи постоянного тока от R0 до R. Предположим, что мы размыкаем контур, когда в нем течет установившийся ток . При размыкании цепи ток изменяется по формуле (127.2). Подставив в нее выражение для

I0 и , получим

Э.д.с. самоиндукции

т. е. при значительном увеличении сопротивления цепи (R/R0>>1), обладающей боль­шой индуктивностью, э.д.с. самоиндукции может во много раз превышать э.д.с. источника тока, включенного в цепь. Таким образом, необходимо учитывать, что контур, содержащий индуктивность, нельзя резко размыкать, так как это (возникнове­ние значительных э.д.с. самоиндукции) может привести к пробою изоляции и выводу из строя измерительных приборов. Если в контур сопротивление вводить постепенно, то э.д.с. самоиндукции не достигнет больших значений.

43. Явление взаимной индукции. Трансформатор.

Рассмотрим два неподвижных контура (1 и 2), которые расположены достаточно близко друг от друга (рис. 1). Если в контуре 1 протекает ток I1, то магнитный поток, который создавается этим током (поле, создающее этот поток, на рисунке изображено сплошными линиями), прямо пропорционален I1. Обозначим через Ф21 часть потока,пронизывающая контур 2. Тогда   (1)  где L21 — коэффициент пропорциональности. 

Рис.1

Если ток I1 меняет свое значение, то в контуре 2 индуцируется э.д.с. ξi2 , которая по закону Фарадея будет равна и противоположна по знаку скорости изменения магнитного потока Ф21, который создается током в первом контуре и пронизыващет второй:    Аналогичным образом, при протекании в контуре 2 тока I2 магнитный поток (его поле изображено на рис. 1 штрихами) пронизывает первый контур. Если Ф12 — часть этого потока, который пронизывает контур 1, то    Если ток I2 меняет свое значение, то в контуре 1 индуцируется э.д.с. ξi1 , которая равна и противоположна по знаку скорости изменения магнитного потока Ф12, который создается током во втором контуре и пронизывает первый:    Явление возникновения э.д.с. в одном из контуров при изменении силы тока в другом называется взаимной индукцией. Коэффициенты пропорциональности L21 и L12 называются взаимной индуктивностью контуров. Расчеты, которые подтверждены опытом, показывают, что L21 и L12 равны друг другу, т. е.   (2)  Коэффициенты пропорциональности L12 и L21 зависят от размеров, геометрической формы, взаимного расположения контуров и от магнитной проницаемости среды, окружающей контуры. Единица взаимной индуктивности та же, что и для индуктивности, — генри (Гн).  Найдем взаимную индуктивность двух катушек, которые намотаны на общий тороидальный сердечник. Этот случай имеет большое практическое значение (рис. 2). Магнитная индукция поля, которое создавается первой катушкой с числом витков N1, током I1 и магнитной проницаемостью μ сердечника, B = μμ0(N1I1/l) где l — длина сердечника по средней линии. Магнитный поток сквозь один виток второй катушки Ф2 = BS = μμ0(N1I1/l)S 

Значит, полный магнитный поток (потокосцепление) сквозь вторичную обмотку, которая содержит N2 витков,    Поток Ψ создается током I1, поэтому, используя (1), найдем   (3)  Если рассчитать магнитный поток, который создавается катушкой 2 сквозь катушку 1, то для L12 получим выражение в соответствии с формулой (3). Значит, взаимная индуктивность двух катушек, которые намотаны на общий тороидальный сердечник,   

Трансформа́тор (от лат. transformo — преобразовывать) — это статическое электромагнитное устройство, имеющее две или более индуктивно связанных обмоток на каком-либо магнитопроводе и предназначенное для преобразования посредствомэлектромагнитной индукции одной или нескольких систем (напряжений) переменного тока в одну или несколько других систем (напряжений) переменного тока без изменения частоты системы (напряжения) переменного тока

формула через силу тока, индуктивность или площадь, единица измерения в физике

Содержание:

  • Что такое ЭДС индукции — когда возникает, при каких условиях
  • ЭДС в быту, как обозначается, единицы измерения
  • Законы Фарадея и Ленца
  • Как рассчитать электродвижущую силу индукции, формулы
    • Через магнитный поток
    • Через силу тока
    • Через сопротивление
    • Через угловую скорость
    • Через площадь

Содержание

  • Что такое ЭДС индукции — когда возникает, при каких условиях
  • ЭДС в быту, как обозначается, единицы измерения
  • Законы Фарадея и Ленца
  • Как рассчитать электродвижущую силу индукции, формулы
    • Через магнитный поток
    • Через силу тока
    • Через сопротивление
    • Через угловую скорость
    • Через площадь

Что такое ЭДС индукции — когда возникает, при каких условиях

Определение

Электродвижущая сила, ЭДС — физическая величина, описывающая работу любых сил, которые действуют в квазистационарных цепях постоянного или переменного тока, за исключением диссипативных и электростатических сил.

При замкнутой цепи можно найти ЭДС, воспользовавшись законом Ома:

\(\varepsilon\;=\;I\;\times\;(R\;+\;r).\)

R здесь — сопротивление цепи, r — внутреннее сопротивление источника.
Создание Алессандро Вольтой надежного источника электричества, гальванического элемента, и открытие Хансом Кристианом Эрстедом магнитного действия электрического тока послужили толчком к интенсивному развитию техники электрических измерений в XIX веке.

Осторожно! Если преподаватель обнаружит плагиат в работе, не избежать крупных проблем (вплоть до отчисления). Если нет возможности написать самому, закажите тут.

Выдающаяся роль здесь принадлежит немецкому физику Георгу Симону Ому. Для определения силы тока он использовал принцип крутильных весов Кулона. На длинной тонкой нити подвешено горизонтальное коромысло с заряженным шариком на конце. Второй заряд закреплен на спице, пропущенной сквозь крышку весов.

При их взаимодействии коромысло поворачивается. Вращение головки в верхней части весов закручивало нить, возвращая коромысло в исходное состояние. По углу закручивания можно рассчитать силу взаимодействия зарядов в зависимости от расстояния между ними.

Ом по величине угла закрутки судил о силе тока I в проводнике, т. е. количестве электричества, перенесенном через поперечное сечение проводника за единицу времени.

В качестве основной характеристики источника тока Ом брал величину напряжения \varepsilon на электродах гальванического элемента при разомкнутой цепи. Эту величину \varepsilon он назвал электродвижущей силой, сокращенно ЭДС.

Движущиеся заряды создают вокруг себя магнитное поле. Однако действующая в нем на магнит или другой ток сила отличается от электрической своим направлением — магнитная стрелка старается развернуться перпендикулярно проводу.

Изучение действующей на другой ток силы переросло в отдельное исследование с неожиданным результатом: сила оказалась направленной всегда перпендикулярно внесенному в магнитное поле проводнику, который для простоты исследования был прямолинейным.

Математическое выражение для этой силы, названной силой Ампера, проще всего записать в виде векторного произведения:

\(d\overrightarrow F\;=\;Id\overrightarrow l\;\times\;\overrightarrow B\).

I здесь — сила тока, протекающего через проводник; l — вектор длины проводника, направленный в ту же сторону, куда течет ток; В — характеристика поля. Величина В называется магнитной индукцией и является аналогом электрической напряженности.

Максвелл поставил целью создать теорию эфира, связав его механические характеристики с электрическими и магнитными силами. Тщательно изучив труды Фарадея, он пришел к выводу, что напряженность \(\overrightarrow Е\) электрического поля объясняется упругими напряжениями в эфире, а магнитная индукция \(\overrightarrow B\) — его вихревыми движениями.

Рассматривая замкнутый проводящий контур С, где действует ЭДС индукции \(\varepsilon_i\), Максвелл для получения числа силовых линий магнитного потока \(\triangle Ф\), пересекаемых контуром за время \triangle t, «натягивал» на него некую поверхность S, разбитую на элементарные площадки \(\triangle S\), и отождествлял Ф с магнитным потоком сквозь всю поверхность. Математически это можно выразить так:

\(Ф\;=\;\sum_{\triangle S}\;\;B\triangle S. \)

Объединив это соотношение с идеей Фарадея, Максвелл пришел к собственной формуле:

\(\varepsilon_i\;=\;-\;\frac1с\;\times\;\frac{dФ}{dt}.\)

Выбор коэффициента пропорциональности \(\alpha\) здесь обусловлен необходимостью согласования формулы с законом Био — Савара — Лапласа, в котором появляется та же электродинамическая постоянная с.

Определение

Электродинамическая постоянная с — универсальная постоянная, равная скорости распространения электромагнитных волн в вакууме.

Но в опытах Фарадея ЭДС индукции регистрировалась как в движущемся, так и в покоящемся проводящем контуре С, если последний находился в переменном магнитном поле. И здесь встал вопрос, что конкретно перемещает заряды в неподвижном проводнике.

Само по себе магнитное поле не воздействует на заряды, находящиеся в покое, из чего следует: условие возникновения индукционного тока — возникающее в контуре электрическое поле \overrightarrow Е.  Так как электростатическое поле в замкнутом контуре не совершает работы, значит, происходит работа вихревого поля, и она равна ЭДС индукции:

\(\varepsilon_i\;=\;\underset С{\oint\;}\;(\overrightarrow{Е\;}\times\;d\overrightarrow l)\)

Определение

Самоиндукция — частный случай магнитной индукции, возникновение ЭДС индукции в проводящем контуре, когда в нем меняется ток.

Источником энергии, возникающей в цепи, является в этом случае запас энергии магнитного поля. Полное количество выделившейся джоулевой теплоты можно вычислить, изобразив на графике зависимость магнитного потока Ф(I) от силы тока I:

Источник: physics.ru

ЭДС в быту, как обозначается, единицы измерения

В быту явление электромагнитной индукции используют для изменения величины напряжения тока в трансформаторах и дросселях. На принципе магнитной индукции работают электрические счетчики, реле мощности, успокоительные системы стрелочных измерительных приборов.

Существуют также магнитные газовые генераторы, в которых благодаря магнитному полю возникает электродвижущая сила, создающая ток.

Электродвижущая сила индукции в системе СИ измеряется в вольтах. Просто электродвижущая сила обозначается греческой буквой \(\varepsilon \), электродвижущая сила индукции —\( \varepsilon_i.\)

Законы Фарадея и Ленца

Фарадей опытным путем выяснил, что при пересечении проводником магнитных силовых линий по нему проходит заряд \(\triangle Q\). Он связан с числом пересеченных силовых линий \( \triangle Ф\) и электрическим сопротивлением контура R, что выражается законом Фарадея:

\(\triangle Q\;=\;\alpha\frac{\triangle Ф}R. \)

Соприкосновение поля и проводника вызвано либо движением проводника, либо изменениями самого магнитного поля. 

Саму электродвижущую силу индукции, связанную с сопротивлением контура и силой тока согласно закону Ома, можно найти по формуле

\(\varepsilon_i\;=\;\alpha\frac{\triangle Ф}{\triangle t}. \)

\(\triangle t\) здесь — время, за которое проходит через поперечное сечение проводника количество электричества \(\triangle Q.\)
Ленц доказал, что индукционный ток всегда направлен так, чтобы противодействовать вызвавшей его причине. Согласно правилу Ленца, в вышеприведенном соотношении следует выбрать отрицательный знак, считая коэффициент\( \alpha \) положительным: 

\(\varepsilon_i\;=\;-\;\alpha\frac{\triangle Ф}{\triangle t}.\)

Как рассчитать электродвижущую силу индукции, формулы

Через магнитный поток

\(\varepsilon_i\;=\;-\;\alpha\frac{\triangle Ф}{\triangle t}. \)

Через силу тока

ЭДС самоиндукции зависит от изменения силы тока, при этом магнитный поток собственного поля через цепь пропорционален току в ней:

\(\varepsilon_{is\;}\;=\;-\;L\frac{\triangle I}{\triangle t}. \)

L здесь — индуктивность проводника.

Через сопротивление

Для ЭДС индукции уравнение закона Ома можно переписать в виде:
\(\varepsilon_{i\;}\;=\;IR\;-\;\varepsilon.\)

Через угловую скорость

\(\varepsilon_i\;=\;В\omega SN\sin\left(\alpha\right). \)

B здесь — индукция магнитного поля, \(\omega\) — угловая скорость вращения рамки, S — площадь рамки, N — число витков, \(\alpha\) — угол между векторами индукции магнитного поля и скорости движения проводника.

Через площадь

Если магнитный поток изменяется без деформации витков, т. е. их количество и площадь не меняются, то можно найти электродвижущую силу индукции через площадь.
Угол \alpha между вектором магнитного поля и нормалью к плоскости витков будет равен:

\(2\mathrm\pi\;\times\;\mathrm v\;\times\;\mathrm t. \)Полный магнитный поток в момент времени t будет равен:

\(\psi_B\;=\;N\;\times\;B\;\times\;S\;\times\;\cos\left(\alpha\right)=\;N\;\times\;B\;\times\;S\;\times\;\cos\left(2\mathrm\pi\;\times\;\mathrm v\;\times\;\mathrm t\right). \)

Тогда \(\varepsilon_i\;=\;-\;\frac{d\psi_B}{dt}=\;2\mathrm{pivNBSsin}\left(2\mathrm{pivt}\right).\)

Насколько полезной была для вас статья?

Рейтинг: 5.00 (Голосов: 1)

Выделите текст и нажмите одновременно клавиши «Ctrl» и «Enter»

Поиск по содержимому

Какая единица служит для измерения эдс индукции

ЭДС. Численно электродвижущая сила измеряется работой, совершаемой источником электрической энергии при переносе единичного положительного заряда по всей замкнутой цепи. Если источник энергии, совершая работу A, обеспечивает перенос по всей замкнутой цепи заряда q, то его электродвижущая сила (Е) будет равна

За единицу измерения электродвижущей силы в системе СИ принимается вольт (в). Источник электрической энергии обладает эдс в 1 вольт, если при перемещении по всей замкнутой цепи заряда в 1 кулон совершается работа, равная 1 джоулю. Физическая природа электродвижущих сил в разных источниках весьма различна.

Самоиндукция – возникновение ЭДС индукции в замкнутом проводящем контуре при изменении тока, протекающего по контуру. При изменении тока I в контуре пропорционально меняется и магнитный поток Bчерез поверхность, ограниченную этим контуром. Изменение этого магнитного потока, в силу закона электромагнитной индукции, приводит к возбуждению в этом контуре индуктивной ЭДС E. Это явление и называется самоиндукцией.

Понятие родственно понятию взаимоиндукции, являясь его частным случаем.

Мощность. Мощность – это работа производимая единицу времени.Мощность-это работа производимая в еденицу времени, т.е для переноса заряда в эл. цепи или в замкнутой затрачивается энергия, которая равна А=U*Q так как кол-во электричества равна произведению силы тока , то Q=I*t отсюда следует что A=U*I*t. P=A/t=U*Q/t=U*I=I*t*R=P=U*I(И)

1Вт=1000мВ, 1кВт=1000В, Pr=Pп+Po-формула баланса мощности. 2*R-теряемая мощность. Для того что бы цепь функционировала необходимо соблюдать баланс мощности в эл.цепи.

12.Закон Ома для участка цепи.

Сила тока в участке цепи прямо пропорциональна напряжению на концах этого проводника и обратно пропорциональна его сопротивлению:
I = U / R; [A = В / Ом]

13.Закон Ома для полной цепи.

Сила тока в цепи пропорциональна действующей в цепи ЭДС и обратно пропорциональна сумме сопротивлений цепи и внутреннего сопротивления источника.

— ЭДС источника напряжения(В), — сила тока в цепи (А), — сопротивление всех внешних элементов цепи(Ом), — внутреннее сопротивление источника напряжения(Ом) .1)E=I(R+r)? 2)R+r=E/I

14.Последовательное, параллельное соединение резисторов, эквивалентное сопротивление. Распределение токов и напряжения.

При последовательном соединении нескольких резисторов конец первого резисторасоединяют с началом второго, конец второго — с началом третьего и т. д. При таком соединении по всем элементам последовательной цепи проходит
один и тот же ток I.

Uэ=U1+U2+U3. Следовательно, напряжение U на зажимах источника равно сумме напряжений на каждом из последовательно включенных резисторов.

Rэ=R1+R2+R3, Iэ=I1=I2=I3, Uэ=U1+U2+U3.

При последовательном соединении сопротивление цепи увеличивается.

Параллельное соединение резисторов. Параллельным соединением сопротивлений называется такое соединение, при котором к одному зажиму источника подключаются начала сопротивлений, а к другому зажиму – концы.

Общее сопротивление параллельно включенных сопротивлений определяется по формуле

Общее сопротивление параллельно включенных сопротивлений всегда меньше наименьшего сопротивления, входящего в данное соединение.

при параллельном соединении сопротивлений напряжения на них равны между собой. Uэ=U1=U2=U3 В цепи притекает ток I, а токи I1, I2, I3 утекают из нее. Так как движущиеся электрические заряды не скапливаются в точке, то очевидно, что суммарный заряд, притекающий к точке разветвления, равен суммарному заряду утекающему от нее:Iэ=I1+I2+I3 Следовательно, третье свойство параллельного соединения может сформулирована так: Величина тока в не разветвленной части цепи равна сумме токов в параллельных ветвях. Для двух парал.резисторов:

Причиной электродвижущей силы может стать изменение магнитного поля в окружающем пространстве. Это явление называетсяэлектромагнитной индукцией. Величина ЭДС индукции в контуре определяется выражением

где — поток магнитного поля через замкнутую поверхность , ограниченную контуром. Знак «−» перед выражением показывает, что индукционный ток, созданный ЭДС индукции, препятствует изменению магнитного потока в контуре (см. правило Ленца).

41. Индуктивность, ее единица СИ. Индуктивность длинного соленоида.

Индукти́вность (или коэффициент самоиндукции) — коэффициент пропорциональности между электрическим током, текущим в каком-либо замкнутом контуре, и магнитным потоком, создаваемым этим током через поверхность [1] , краем которой является этот контур. [2][3][4] .

— магнитный поток, — ток в контуре, — индуктивность.

Нередко говорят об индуктивности прямого длинного провода(см.). В этом случае и других (особенно – в не отвечающих квазистационарному приближению) случаях, когда замкнутый контур непросто адекватно и однозначно указать, приведенное выше определение требует особых уточнений; отчасти полезным для этого оказывается подход (упоминаемый ниже), связывающий индуктивность с энергией магнитного поля.

Через индуктивность выражается ЭДС самоиндукции в контуре, возникающая при изменении в нём тока [4] :

.

Из этой формулы следует, что индуктивность численно равна ЭДС самоиндукции, возникающей в контуре при изменении силы тока на 1 А за 1 с.

При заданной силе тока индуктивность определяет энергию магнитного поля, создаваемого этим током [4] :

.

Обозначение и единицы измерения

В системе единиц СИ индуктивность измеряется в генри [7] , сокращенно Гн, в системе СГС — в сантиметрах (1 Гн = 10 9 см) [4] . Контур обладает индуктивностью в один генри, если при изменении тока на один ампер в секунду на выводах контура будет возникать напряжение в один вольт. Реальный, не сверхпроводящий, контур обладает омическим сопротивлением R, поэтому на нём будет дополнительно возникать напряжение U=I*R, где I — сила тока, протекающего по контуру в данное мгновение времени.

Символ , используемый для обозначения индуктивности, был взят в честь Ленца Эмилия Христиановича (Heinrich Friedrich Emil Lenz) [ источник не указан 1017 дней ] . Единица измерения индуктивности названа в честь Джозефа Генри (Joseph Henry) [8] . Сам термин индуктивность был предложен Оливером Хевисайдом (Oliver Heaviside) в феврале 1886 года [ источник не указан 1017 дней ] .

Электрический ток, который течет в замкнутом контуре, создает вокруг себя магнитное поле, индукция которого, согласно закону Био-Савара-Лапласа, пропорциональна току. Сцепленный с контуром магнитный поток Ф поэтому прямо пропорционален току I в контуре: (1) где коэффициент пропорциональности L называетсяиндуктивностью контура. При изменении в контуре силы тока будет также изменяться и сцепленный с ним магнитный поток; значит, в контуре будет индуцироваться э.д.с. Возникновение э.д.с. индукции в проводящем контуре при изменении в нем силы тока называетсясамоиндукцией. Из выражения (1) задается единица индуктивности генри (Гн): 1 Гн — индуктивность контура, магнитный поток самоиндукции которого при токе в 1 А равен 1 Вб: 1 Гн = 1 Вб/с = 1 В

Вычислим индуктивность бесконечно длинного соленоида. Полный магнитный поток сквозь соленоид (потокосцепление) равен μμ(N 2 I/l)S . Подставив в (1), найдем (2) т. е. индуктивность соленоида зависит от длиныl солениода, числа его витков N, его , площади S и магнитной проницаемости μ вещества, из которого изготовлен сердечник соленоида. Доказано, что индуктивность контура зависит в общем случае только от геометрической формы контура, его размеров и магнитной проницаемости среды, в которой он расположен, и можно провести аналог индуктивности контура с электрической емкостью уединенного проводника, которая также зависит только от формы проводника, его размеров и диэлектрической проницаемости среды. Найдем, применяя к явлению самоиндукции закон Фарадея, что э.д.с. самоиндукции равна Если контур не претерпевает деформаций и магнитная проницаемость среды остается неизменной (в дальнейшем будет показано, что последнее условие выполняется не всегда), то L = const и(3) где знак минус, определяемый правилом Ленца, говорит о том, чтоналичие индуктивности в контуре приводит к замедлению изменения тока в нем. Если ток со временем увеличивается, то (dI/dt 0 т. е. ток самоиндукции направлен навстречу току, обусловленному внешним источником, и замедляет его увеличение. Если ток со временем уменьшается, то (dI/dt>0) и ξs >1), обладающей боль­шой индуктивностью, э.д.с. самоиндукции может во много раз превышать э.д.с. источника тока, включенного в цепь. Таким образом, необходимо учитывать, что контур, содержащий индуктивность, нельзя резко размыкать, так как это (возникнове­ние значительных э.д.с. самоиндукции) может привести к пробою изоляции и выводу из строя измерительных приборов. Если в контур сопротивление вводить постепенно, то э.д.с. самоиндукции не достигнет больших значений.

43. Явление взаимной индукции. Трансформатор.

Рассмотрим два неподвижных контура (1 и 2), которые расположены достаточно близко друг от друга (рис. 1). Если в контуре 1 протекает ток I1, то магнитный поток, который создавается этим током (поле, создающее этот поток, на рисунке изображено сплошными линиями), прямо пропорционален I1. Обозначим через Ф21 часть потока,пронизывающая контур 2. Тогда (1) где L21 — коэффициент пропорциональности.

Если ток I1 меняет свое значение, то в контуре 2 индуцируется э.д.с. ξi2 , которая по закону Фарадея будет равна и противоположна по знаку скорости изменения магнитного потока Ф21, который создается током в первом контуре и пронизыващет второй: Аналогичным образом, при протекании в контуре 2 тока I2 магнитный поток (его поле изображено на рис. 1 штрихами) пронизывает первый контур. Если Ф12 — часть этого потока, который пронизывает контур 1, то Если ток I2 меняет свое значение, то в контуре 1 индуцируется э.д.с. ξi1 , которая равна и противоположна по знаку скорости изменения магнитного потока Ф12, который создается током во втором контуре и пронизывает первый: Явление возникновения э.д.с. в одном из контуров при изменении силы тока в другом называется взаимной индукцией. Коэффициенты пропорциональности L21 и L12 называются взаимной индуктивностью контуров. Расчеты, которые подтверждены опытом, показывают, что L21 и L12 равны друг другу, т. е. (2) Коэффициенты пропорциональности L12 и L21 зависят от размеров, геометрической формы, взаимного расположения контуров и от магнитной проницаемости среды, окружающей контуры. Единица взаимной индуктивности та же, что и для индуктивности, — генри (Гн). Найдем взаимную индуктивность двух катушек, которые намотаны на общий тороидальный сердечник. Этот случай имеет большое практическое значение (рис. 2). Магнитная индукция поля, которое создавается первой катушкой с числом витков N1, током I1 и магнитной проницаемостью μ сердечника, B = μμ(N1I1/l) где l — длина сердечника по средней линии. Магнитный поток сквозь один виток второй катушки Ф2 = BS = μμ(N1I1/l)S

Значит, полный магнитный поток (потокосцепление) сквозь вторичную обмотку, которая содержит N2 витков, Поток Ψ создается током I1, поэтому, используя (1), найдем (3) Если рассчитать магнитный поток, который создавается катушкой 2 сквозь катушку 1, то для L12 получим выражение в соответствии с формулой (3). Значит, взаимная индуктивность двух катушек, которые намотаны на общий тороидальный сердечник,

Трансформа́тор (от лат. transformo — преобразовывать) — это статическое электромагнитное устройство, имеющее две или более индуктивно связанных обмоток на каком-либо магнитопроводе и предназначенное для преобразования посредствомэлектромагнитной индукции одной или нескольких систем (напряжений) переменного тока в одну или несколько других систем (напряжений) переменного тока без изменения частоты системы (напряжения) переменного тока

В материале разберемся в понятии ЭДС индукции в ситуациях ее возникновения. Также рассмотрим индуктивность в качестве ключевого параметра возникновения магнитного потока при появлении электрического поля в проводнике.

Электромагнитная индукция представляет собой генерирование электрического тока магнитными полями, которые изменяются во времени. Благодаря открытиям Фарадея и Ленца закономерности были сформулированы в законы, что ввело симметрию в понимание электромагнитных потоков. Теория Максвелла собрала воедино знания об электрическом токе и магнитных потоках. Благодаря открытия Герца человечество узнало о телекоммуникациях.

Магнитный поток

Вокруг проводника с электротоком появляется электромагнитное поле, однако параллельно возникает также обратное явление – электромагнитная индукция. Рассмотрим магнитный поток на примере: если рамку из проводника поместить в электрическое поле с индукцией и перемещать ее сверху вниз по магнитным силовым линиям или вправо-влево перпендикулярно им, тогда магнитный поток, проходящий через рамку, будет постоянной величиной.

При вращении рамки вокруг своей оси, тогда через некоторое время магнитный поток изменится на определенную величину. В результате в рамке возникает ЭДС индукции и появится электрический ток, который называется индукционным.

ЭДС индукции

Разберемся детально, что такое понятие ЭДС индукции. При помещении в магнитное поле проводника и его движении с пересечением силовых линий поля, в проводнике появляется электродвижущая сила под названием ЭДС индукции. Также она возникает, если проводник остается в неподвижном состоянии, а магнитное поле перемещается и пересекается с проводником силовыми линиями.

Когда проводник, где происходит возникновение ЭДС, замыкается на вешнюю цепь, благодаря наличию данной ЭДС по цепи начинает протекать индукционный ток. Электромагнитная индукция предполагает явление индуктирования ЭДС в проводнике в момент его пересечения силовыми линиями магнитного поля.

Электромагнитная индукция являет собой обратный процесс трансформации механической энергии в электроток. Данное понятие и его закономерности широко используются в электротехнике, большинство электромашин основывается на данном явлении.

Законы Фарадея и Ленца

Законы Фарадея и Ленца отображают закономерности возникновения электромагнитной индукции.

Фарадей выявил, что магнитные эффекты появляются в результате изменения магнитного потока во времени. В момент пересечения проводника переменным магнитным током, в нем возникает электродвижущая сила, которая приводит к возникновению электрического тока. Генерировать ток может как постоянный магнит, так и электромагнит.

Ученый определил, что интенсивность тока возрастает при быстром изменении количества силовых линий, которые пересекают контур. То есть ЭДС электромагнитной индукции пребывает в прямой зависимости от скорости магнитного потока.

Согласно закону Фарадея, формулы ЭДС индукции определяются следующим образом:

Знак «минус» указывает на взаимосвязь между полярностью индуцированной ЭДС, направлением потока и изменяющейся скоростью.

Согласно закону Ленца, можно охарактеризовать электродвижущую силу в зависимости от ее направленности. Любое изменение магнитного потока в катушке приводит к появлению ЭДС индукции, причем при быстром изменении наблюдается возрастающая ЭДС.

Если катушка, где есть ЭДС индукции, имеет замыкание на внешнюю цепь, тогда по ней течет индукционный ток, вследствие чего вокруг проводника появляется магнитное поле и катушка приобретает свойства соленоида. В результате вокруг катушки формируется свое магнитное поле.

Э.Х. Ленц установил закономерность, согласно которой определяется направление индукционного тока в катушке и ЭДС индукции. Закон гласит, что ЭДС индукции в катушке при изменении магнитного потока формирует в катушке ток направления, при котором данный магнитный поток катушки дает возможность избежать изменения постороннего магнитного потока.

Закон Ленца применяется для всех ситуаций индуктирования электротока в проводниках, вне зависимости от их конфигурации и метода изменения внешнего магнитного поля.

Движение провода в магнитном поле

Значение индуктированной ЭДС определяется в зависимости от длины проводника, пересекаемого силовыми линиями поля. При большем количестве силовых линий возрастает величина индуктируемой ЭДС. При увеличении магнитного поля и индукции, большее значение ЭДС возникает в проводнике. Таким образом, значение ЭДС индукции в движущемся в магнитном поле проводнике находится в прямой зависимости от индукции магнитного поля, длины проводника и скорости его движения.

Данная зависимость отражена в формуле Е = Blv, где Е — ЭДС индукции; В — значение магнитной индукции; I — длина проводника; v —скорость его перемещения.

Отметим, что в проводнике, который движется в магнитном поле, ЭДС индукции появляется, только когда он пересекает силовые линии магнитного поля. Если проводник движется по силовым линиям, тогда ЭДС не индуктируется. По этой причине формула применяется только в случаях, когда движением проводника направлено перпендикулярно силовым линиям.

Направление индуктированной ЭДС и электротока в проводнике определяется направлением движения самого проводника. Для выявления направления разработано правило правой руки. Если держать ладонь правой руки таким образом, чтобы в ее направлении входили силовые линии поля, а большой палец указывает направление движения проводника, тогда остальные четыре пальца показывают направление индуктированной ЭДС и направление электротока в проводнике.

Вращающаяся катушка

Функционирование генератора электротока основывается на вращении катушки в магнитном потоке, где имеется определенное количество витков. ЭДС индуцируется в электрической цепи всегда при пересечении ее магнитным потоком, на основании формулы магнитного потока Ф = B x S х cos α (магнитная индукция, умноженная на площадь поверхности, через которую проходит магнитный поток, и косинус угла, сформированный вектором направления и перпендикулярной плоскости линии).

Согласно формуле, на Ф воздействуют изменения в ситуациях:

  • при изменении магнитного потока меняется вектор направления;
  • изменяется площадь, заключенная в контур;
  • меняется угол.

Допускается индуцирование ЭДС при неподвижном магните или неизменном токе, а просто при вращении катушки вокруг своей оси в пределах магнитного поля. В данном случае магнитный поток изменяется при смене значения угла. Катушка в процессе вращения пересекает силовые линии магнитного потока, в итоге появляется ЭДС. При равномерном вращении возникает периодическое изменение магнитного потока. Также число силовых линий, которые пересекаются ежесекундно, становится равным значениям через равные временные промежутки.

На практике в генераторах переменного электротока катушка остается в неподвижном состоянии, а электромагнит выполняет вращения вокруг нее.

ЭДС самоиндукции

При прохождении через катушку переменного электротока генерируется переменное магнитное поле, которое характеризуется меняющимся магнитным потоком, индуцирующим ЭДС. Данное явление называется самоиндукцией.

В силу того, что магнитный поток пропорционален интенсивности электротока, тогда формула ЭДС самоиндукции выглядит таким образом:

Ф = L x I, где L – индуктивность, которая измеряется в Гн. Ее величина определяется числом витков на единицу длины и величиной их поперечного сечения.

Взаимоиндукция

При расположении двух катушек рядом в них наблюдается ЭДС взаимоиндукции, которая определяется конфигурацией двух схем и их взаимной ориентацией. При возрастании разделения цепей значение взаимоиндуктивности уменьшается, поскольку наблюдается уменьшение общего для двух катушек магнитного потока.

Рассмотрим детально процесс возникновения взаимоиндукции. Есть две катушки, по проводу одной с N1 витков течет ток I1, которым создается магнитный поток и идет через вторую катушку с N2 числом витков.

Значение взаимоиндуктивности второй катушки в отношении первой:

М21 = (N2 x F21)/I1.

Значение магнитного потока:

Ф21 = (М21/N2) x I1.

Индуцированная ЭДС вычисляется по формуле:

Е2 = — N2 x dФ21/dt = — M21x dI1/dt.

В первой катушке значение индуцируемой ЭДС:

Е1 = — M12 x dI2/dt.

Важно отметить, что электродвижущая сила, спровоцированная взаимоиндукцией в одной из катушек, в любом случае прямо пропорциональна изменению электрического тока в другой катушке.

Тогда взаимоиндуктивность считается равной:

Вследствие этого , E1 = — M x dI2/dt и E2 = M x dI1/dt. М = К √ (L1 x L2), где К является коэффициентом связи между двумя значениями инжуктивности.

Взаимоиндукция широко используется в трансформаторах, которые дают возможность менять значения переменного электротока. Прибор представляет собой пару катушек, которые намотаны на общий сердечник. Ток в первой катушке формирует изменяющийся магнитный поток в магнитопроводе и ток во второй катушке. При меньшем числе витков в первой катушке, чем во второй, возрастает напряжение, и соответственно при большем количестве витков в первой обмотке напряжение снижается.

Помимо генерирования и трансформации электрической энергии, явление магнитной индукции используется в прочих приборах. К примеру, в магнитных левитационных поездах, движущихся без непосредственного контакта с током в рельсах, а на пару сантиметров выше по причине электромагнитного отталкивания.

Электродвижущая сила индукции

Электрические токи порождают вокруг себя магнитные поля. Данная связь дала толчок к многочисленным попыткам создать электрический ток в контуре при помощи магнитного поля.

Данную задачу решил М. Фарадей в 1831 году. Ученый открыл явление электромагнитной индукции.

Электромагнитная индукция

Явление электромагнитной индукции заключается в том, что в замкнутом проводящем контуре, если изменяется поток магнитной индукции, который рассматриваемый контур охватывает, появляется электрический ток. Возникающий электрический ток называют током индукции.

Анализируя свои множественные эксперименты, М. Фарадей пришел к выводу о том, что:

  1. Индукционный ток появляется всегда при изменении магнитного потока, который охватывает проводящий контур. Так, если в однородном магнитном поле проводящий замкнутый контур повернуть, то в момент разворота в нем будет течь ток индукции. В этом случае индукция магнитного поля постоянна около проводящего контура, переменным является только поток магнитной индукции, который изменяется за счет изменения площади контура.
  2. Величина тока индукции не связана со способом изменения магнитного потока. Она определена только скоростью его изменения. Сила тока индукции тем больше, чем больше скорость перемещения магнита, или быстрота изменения силы тока, или скорость перемещения катушек.

Электромагнитная индукция подтверждает связь между электрическими и магнитными явлениями.

Закон Фарадея

Анализируя данные своих экспериментов, М. Фарадей предложил количественный закон, описывающий электромагнитную индукцию. Ученый доказал, что каждый раз при изменении потока магнитной индукции, который сцеплен с проводящим контуром, в проводнике появляется ток индукции. Наличие индукционного тока означает то, что в цепи присутствует электродвижущая сила (ЭДС), которую в данном случае называют электродвижущей силой электромагнитной индукции ($Ɛ_i$).

Величина тока индукции, а значит, и величина $Ɛ_i$ зависит от скорости изменения магнитного потока:

$\left|Ɛ_{i} \right|=\frac{dФ}{dt}\left( 1 \right)$.

где $Ф$ – поток магнитной индукции.

Определимся со знаком ЭДС индукции. Знак потока магнитной индукции связан с выбором положительной нормали к рассматриваемому проводящему контуру. А направление силы тока и направление нормали связывает правило правого буравчика (винта). Получается, что фиксируя направление нормали, мы устанавливаем знак магнитного потока, направление тока и $Ɛ_i$ в контуре.

Сформулируем закон электромагнитной индукции Фарадея в окончательном виде:

Определение 1

Не зависимо от причины изменения магнитного потока, который охватывает замкнутый проводящий контур, электродвижущая сила индукции, появляющаяся в этом контуре равна:

$Ɛ_{i}=-\frac{dФ}{dt}\left( 2 \right)$.

где под $\frac{dФ}{dt}$ понимают полную скорость изменения потока магнитной индукции, охватываемого проводником.

Минус в формуле (2) указывает на то, что:

  • При росте потока магнитной индукции (скорость изменения магнитного потока больше нуля) ($\frac{dФ}{dt}>0)$, ЭДС индукции меньше нуля ($Ɛ_i
  • При уменьшении потока магнитной индукции (скорость изменения магнитного потока меньше нуля), ЭДС индукции больше нуля ($Ɛ_i>0$). Что значит, направление потока и направление поля тока индукции совпадают.

Замечание 1

Знак минус в формуле (2) – это математическое отображение правила Ленца, которое используют для того, чтобы найти направление тока индукции.

Закон Фарадея справедлив при:

  1. произвольных перемещениях замкнутого проводящего контура;
  2. при любых его деформациях;
  3. изменениях магнитного поля.

ЭДС индукции измеряется с Международной системе единиц (СИ) в вольтах (В).

$\left[ Ɛ_{i} \right]=\left[ \frac{dФ}{dt} \right]=\frac{Вб}{с}$=В.

Значение закона Фарадея

Закон Фарадея выражает новое физическое явление, в котором переменное магнитное поле порождает электрическое поле. Отсюда делается вывод о том, что электрическое поле может порождаться не только электрическими зарядами, но и изменяющимся магнитным полем.

Электромагнитная индукция – это всеобщий фундаментальный закон природы, реализующий связь между электрическими и магнитными полями.

Природа электродвижущей силы индукции

Если проводник перемещается в магнитном поле, то на свободные электроны его вещества действуют силы Лоренца. Эти электроны под воздействием названной силы приходят в движение относительно проводника, что означает: в проводнике появляется ток.

Рисунок 1. Проводники. Автор24 — интернет-биржа студенческих работ

Рассмотрим прямой участок $DG$ проводника на рис.1. Этот участок перемещается со скоростью $\vec v$ по проводникам $CK$ и $AL$, как по направляющим. При этом контур $AGDCA$ постоянно замкнут. Вектор индукции внешнего магнитного поля нормален плоскости рассматриваемого контура. Магнитное поле будем считать однородным. На заряды, которые перемещаются вместе с проводником, действует сила Лоренца, равная:

$\vec{F}_{L}=q\left( \vec{v}\times \vec{B} \right)\left( 3 \right)$. D {vBdl=vBl\, \left( 6 \right).} $

На не движущихся участках замкнутого контура, который мы рассматриваем, ЭДС не возникает. Следовательно, ЭДС контура равна ЭДС подвижного проводника $DG$, перемещающейся в магнитном поле.

$Ɛ_{i}=\int\limits_{AGDCA} {\vec{E}d\vec{l}=vBl\, \left( 7 \right).} $

Скорость перемещения проводника выразим как:

$v=\frac{dx}{dt}\left( 8 \right)$.

где $x$ – координата контактов проводника в точках $D$ и $G$ направляющими проводниками:

$Ɛ_{i}=Bl\frac{dx}{dt}\left( 9 \right)$.

Учитывая, что:

$Ф=-xlB$ (10),

где $Ф$ – поток магнитной индукции через поверхность, которую ограничивает контур $AGDCA$. Знак минус указывает на то, что направления векторов $\vec B$ и $ d\vec S$ противоположны,

окончательно имеем:

$Ɛ_{i}=-\frac{dФ}{dt}\left( 11 \right)$.

Выражение (11) мы получили, рассматривая движение части проводника. При перемещении нескольких участков проводника, ЭДС индукции находят как алгебраическую сумму ЭДС индукции, появляющихся на каждом участке.

Единица измерения магнитной индукции, теория и онлайн калькуляторы

Единица измерения магнитной индукции, теория и онлайн калькуляторы

Тесла – единица измерения магнитной индукции в системе СИ

Единица магнитной индукции ($\overline{B}$) в международной системе единиц (СИ) называется тесла (Тл), по имени сербского ученого Н. Тесла, который успешно работал в области радиотехники и электроники.

Единицу измерения магнитной индукции определим исходя из закона Ампера. Рассмотрим прямолинейный проводник, длиной $l$ по которому течет ток $I$. Пусть этот проводник находится в однородном магнитном поле $\overline{B}$, причем вектор индукции поля перпендикулярен проводнику. В таком случае модуль силы Ампера (${\overline{F}}_A$), воздействующей на проводник равен:

\[F=IBl\ \left(1\right).\]

Выразим магнитную индукцию из формулы (1), получим:

\[B=\frac{F}{I\cdot l}\left(2\right).\]

Из выражения (2) мы видим, что тесла (единица измерения магнитной индукции) – это величина, соответствующая магнитной индукции однородного магнитного поля, действующего на каждый метр прямого проводника, находящегося в магнитном поле перпендикулярно направлению $\overline{B}$, с силой в один ньютон, при силе тока в проводнике в один ампер:

\[\left[B\right]=Тл=\frac{H}{A\cdot м}. 4Гс.\]

Данная единица измерения именована в честь немецкого ученого К.Ф. Гаусса.

Используя основные единицы системы СГС, единица измерения магнитной индукции выражается как:

\[\left[B\right]=\frac{\sqrt{гр}}{с\cdot \sqrt{см}}.\]

Примеры задач с решением

Пример 1

Задание. Получите единицу измерения магнитной индукции в Международной системе единиц, используя формулу, связывающую ее с магнитным потоком ($Ф$).

Решение. По условию задачи в качестве основы для ее решения используем выражение:

\[Ф=BS{\cos \alpha \ }\ \left(1.1\right),\]

где$\ Ф$- поток вектора магнитной индукции через площадку S;$\ S$ – величина площади площадки; $\alpha $ – угол между направлением нормали к площади S и направлением вектора магнитной индукции. Выразим модуль вектора магнитной индукции из формулы (1.1), имеем:

\[B=\frac{Ф}{S\ cos\ \alpha }\]

Учитывая, что в системе СИ ${\cos \alpha \ }$ – величина безразмерная, поток вектора магнитной индукции измеряется в веберах (Вб):

\[\left[Ф\right]=Вб=\frac{кг\cdot м^2}{А\cdot с^2},\]

а единицы измерения площади:

\[\left[S\right]=м^2,\]

получим:

\[\left[B\right]=\frac{Вб}{м^2}=\frac{кг\cdot м^2}{А\cdot с^2}\cdot \frac{1}{м^2}=\frac{кг}{А\cdot с^2}=Тл. 2\cdot А}=Тл.\]

Ответ. Мы получили, что тесла можно выразить как: $Тл=\frac{Н}{А\cdot м}$

Читать дальше: единица измерения мощности.

236

проверенных автора готовы помочь в написании работы любой сложности

Мы помогли уже 4 396 ученикам и студентам сдать работы от решения задач до дипломных на отлично! Узнай стоимость своей работы за 15 минут!

Магнитная индукция. Что такое магнитная индукция

Автор Historian Просмотров 32 Опубликовано

ЭМС измеряется в вольтах. Но скорость изменения магнитного потока также измеряется в вольтах! Действительно, из (3) видно, что Vb / s = V. Таким образом, единицы измерения обеих частей пропорциональности (4) совпадают, поэтому коэффициент пропорциональности безразмерный. В системе СИ он принимается равным 1 и составляет

Содержание

  1. Электромагнитная индукция
  2. Магнитный поток
  3. ЭДС индукции
  4. Магнитная индукция
  5. Формулы вычисления магнитной индукции
  6. Формула магнитной индукции:
  7. Другие формулы, где встречается B
  8. Сила Ампера:
  9. Сила Лоренца:
  10. Магнитный поток:
  11. Электромагнитная индукция и магнитная индукция: какая между ними разница?
  12. Физический смысл
  13. Формула и обозначения
  14. Магнитное поле
  15. B и H магнитная индукция

Электромагнитная индукция

Эксперименты Элстеда показали, что электрические токи генерируют магнитные поля в окружающем пространстве. У Майкла Фарадея возникла идея, что может быть и обратный эффект: магнитное поле производит электричество.

Другими словами, даже если проводник замкнут в магнитном поле, может ли действие магнитного поля не производить электричество в этом проводнике?

После десяти лет исследований и экспериментов Фарадей, наконец, обнаружил это явление. В 1831 году он провел следующие эксперименты.

1. две катушки были намотаны на одном деревянном основании; обмотка второй катушки была помещена между витками первой катушки и изолирована. Кабель первой катушки был подключен к источнику питания, а кабель второй катушки — к токоизмерительному клещу (токоизмерительный клещ — это высокочувствительный прибор для измерения малых токов). Поэтому были созданы две цепи: «источник тока — первая катушка» и «вторая катушка — измеритель тока».

Между контурами не было электрического контакта, только магнитное поле первой катушки проникало во вторую катушку.

Когда первая катушка была замкнута, гальванометр регистрировал короткий слабый импульс тока во второй катушке.

Когда по первой катушке протекал постоянный ток, во второй катушке ток не возникал.

Когда цепь первой катушки размыкалась, во второй катушке снова возникал короткий слабый импульс тока, но на этот раз в направлении, противоположном направлению тока при замыкании цепи.

Меняющееся во времени магнитное поле первой катушки создает (или, как говорят, вызывает) ток во второй катушке. Этот ток называется индуцированным током.

По мере увеличения магнитного поля первой катушки (по мере увеличения тока при замыкании цепи) индукционный ток во второй катушке течет в том же направлении.

Если магнитное поле первой катушки уменьшается (когда ток уменьшается при разомкнутой цепи), индукционный ток во второй катушке течет в противоположном направлении.

Если магнитное поле первой катушки остается неизменным (постоянный ток через нее), то во второй катушке нет индуцированного тока.

Открытое им явление Фарадей назвал электромагнитной индукцией («индукция электричества магнетизмом»).

2. Для подтверждения своего предположения о том, что индукционные токи вызываются переменными магнитными полями, Фарадей сдвинул катушки друг относительно друга. Цепь первой катушки всегда была замкнута и находилась под постоянным током, но при движении (приближении или удалении) вторая катушка оказывалась в переменном магнитном поле первой катушки.

Гальванометр снова зарегистрировал ток во второй катушке. Индуцированный ток был в одном направлении, когда катушки находились близко друг к другу, и в другом направлении, когда катушки раздвигались. Чем быстрее двигались катушки, тем больше был индукционный ток.

3. первая катушка была заменена постоянным магнитом. Когда магнит перемещался во второй катушке, возникал индукционный ток. Когда магнит был вытащен, ток снова появился, но в противоположном направлении. Опять же, чем быстрее двигался магнит, тем больше был индукционный ток.

Эти и последующие эксперименты показали, что каждое изменение «числа линий» магнитного поля, проходящего через контур, порождает индукционный поток в проводящей цепи. Чем сильнее индуцированный ток, тем быстрее изменяется это число линий. Направление тока будет в сторону увеличения числа линий, проходящих через цепь, и уменьшения числа других линий.

Магнитный поток

Понятие магнитного тока — это именно характеристика числа линий магнитного поля, проходящих через контур.

Для простоты мы ограничиваемся случаем однородного магнитного поля. Рассмотрим контур области в магнитном поле с индукцией.

Предположим, что магнитное поле изначально перпендикулярно уровню контура (рис. 1).

В этом случае магнитный поток определяется очень просто.

Теперь рассмотрим общий случай, когда вектор образует угол с уровнем нормали контура (рис. 2).

Теперь мы видим, что только перпендикулярная составляющая вектора магнитной индукции «течет» через контур (и никакая составляющая, параллельная контуру, не «течет» через него). Таким образом, в соответствии с типом (1) имеем. Однако, следовательно.

Это общее определение магнитного потока в случае однородного магнитного поля. Обратите внимание, что если вектор параллелен уровню контура (т.е.), то магнитный поток сбрасывается.

Но как определяется магнитный поток, если поле неоднородно? Позвольте нам дать вам представление. Поверхность петли разделена на такое количество очень маленьких областей, что поле считается однородным. Для каждого региона рассчитайте свой малый магнитный поток по типу (2) и сложите все эти магнитные потоки.

Магнитные поточные устройства — Вебера (VB). Как мы увидим.

Почему же магнитный ток характеризует «количество линий» магнитного поля, пронизывающего контур? Все очень просто. Количество линий» определяется их плотностью (и, следовательно, размером — чем больше индукция, тем плотнее линии, и «эффективной» поверхностью, пронизываемой полем (что не одно и то же). Но что именно представляет собой мультипликатор и магнитный ток!

Это дает четкое определение явления электромагнитной индукции, открытого Фарадеем.

Электромагнитная индукция — это явление возникновения электричества в замкнутом проводнике за счет изменения магнитного тока, пронизывающего контур.

ЭДС индукции

Каков механизм индуцированных токов? Это будет объяснено позже. В настоящее время ясно одно. Когда магнитный ток через цепь изменяется, некоторые силы действуют на свободный груз в цепи, т.е. посторонние силы, вызывающие движение груза.

Как известно, работа посторонних сил по перемещению отдельных положительных грузов по контуру называется мощностью (HED). В нашем случае, когда магнитный ток через контур изменяется, соответствующая ЭЭД называется индуктивной индукцией и обозначается.

Таким образом, индуктивный ВЭД — это работа посторонних сил, которая возникает при изменении магнитного тока в цепи и перемещении единичного положительного груза по цепи.

Вскоре мы узнаем природу экзогенных сил, которые возникают в цепи в этом случае.

Стоит отметить, что выражение для индуктивного HED (7) также может быть получено с помощью метода Фарадея. Давайте сделаем это. Со временем наш стержень проходит через корень и занимает свое место (рис. 9). Площадь контура увеличивается по цене прямоугольной площади.

Магнитная индукция

Магнитная индукция — это сила, которая характеризует магнитное поле в выбранных точках пространства. Она определяет силу, оказываемую магнитным полем на движущиеся в нем заряженные частицы. Магнитная индукция рассматривается как фундаментальное свойство магнитного поля (например, напряженность электрического поля).

Магнитная индукция представляет собой магнитную силу (вектор), действующую на пробный объект (например, кусок железа) в любой точке пространства. Проще говоря, когда природные магниты приближаются к магнитным материалам (например, железу, никелю, кобальту), они создают магнитные свойства, известные как «магнитная индукция». Магнитная индукция используется для создания искусственных магнитов.

Магнитную индукцию также называют плотностью магнитного потока.

Магнитная индукция измеряется:.

Отношение тл к гс: 1 тл = 10 000 гс.

Магнитная индукция является векторной величиной и обозначается буквой B со стрелкой.

Индукция (от лат. induction — введение, причина) — это возникновение тока в цепи под действием магнита или другого тока.

Формулы вычисления магнитной индукции

Формула магнитной индукции:

Формула для магнитной индукции: B=Mmax/ IS

  • B — индукция магнитного поля (в Тл)
  • Mmax — максимальный крутящий момент магнитных сил, приложенных к рамке (в Нм)
  • l — длина проводника (в м)
  • S — площадь рамки (в м²)

Другие формулы, где встречается B

Эти уравнения также могут быть использованы для расчета

Сила Ампера:

Сила Ампера: Fa = IBL sina

  • Fa — сила Ампера (в Н — ньютон)
  • I — сила тока (в А — ампер)
  • B — индукция магнитного поля (в Тл)
  • L — длина проводника (в м)
  • α — угол между вектором В и одним из направлений (силы тока, скорости или др.; измеряется в рад. или град.)
Сила Лоренца:

Сила Лоренца: Fl = qvBsinα

  • Fл — сила Лоренца (в Н — ньютон)
  • q — заряд частицы (в Кл — кулон)
  • v — скорость (в м/с)
  • B — индукция (в Тл)
  • α — угол между вектором В и одним из направлений (силы тока, скорости, или др. ; измеряется в рад. или град.))
Магнитный поток:

Магнитный поток: F = BScosα

  • Ф — магнитный поток (в Вб — вебер)
  • B — индукция (в Тл)
  • S — площадь рамки (в м²)
  • α — угол между вектором В и одним из направлений (силы тока, скорости, или др.; измеряется в рад. или град.))

Электромагнитная индукция и магнитная индукция: какая между ними разница?

Электромагнитная индукция — это генерация электродвижущей силы, возникающей в результате относительного движения между магнитным полем и проводником.

Магнитная индукция может создавать или не создавать постоянный магнит.

Электромагнитная индукция производит ток, но этот производимый ток компенсируется изменениями в магнитном поле.

Электромагнитная индукция использует магнит и электрическую цепь, в то время как магнитная индукция использует только магнит и магнитные материалы.

Поэтому поместите цепь в магнитное поле. Определите направление положительного шунтирования цепи. Предположим, что магнитное поле направлено туда, если смотреть с места положительного обхода против часовой стрелки. В этом случае магнитный поток будет положительным: 0’alt =’\ Phi>0’/>.

Физический смысл

С научной точки зрения этот феномен можно объяснить следующим образом. В основе каждого металла лежит кристаллическая решетка. Кристаллическая решетка содержит отрицательно заряженные частицы (электроны). Когда проводник свободен от внешних магнитных воздействий, заряженные частицы полностью неподвижны.

Однако, когда проводник подвергается воздействию магнитных полей в разных направлениях, эти частицы начинают двигаться. Аппарат, используемый в лаборатории для создания магнитных полей и наблюдения индукционных явлений, состоит из металлической катушки и движущегося в ней постоянного магнита. В результате движения внутри металла возникает электрический ток. Сила тока, производимого катушкой, зависит от нескольких факторов

  1. Свойств металла, из которого сделана катушка.
  2. Свойств магнита, перемещающегося внутри катушки.
  3. Скорости движения катушки и магнита относительно друг друга.

В результате воздействия динамического магнитного поля магнита на кристаллическую решетку катушки, электроны в катушке выравниваются вдоль направления динамических линий магнитного поля и поворачиваются на определенный угол.

И чем сильнее магнитное поле, тем большее количество электронов вращается внутри металла. Их положение в кристаллической решетке становится более равномерным. Магнитные поля отдельных частиц не нейтрализуют друг друга, а усиливают друг друга, образуя единое магнитное поле.

Формула и обозначения

Магнитная индукция обозначается латинским символом «B». Она также определяет интенсивность внешнего воздействия магнитного поля на заряженную частицу — в нашем случае обозначенную буквой «Q», в конкретный момент времени. Скорость заряженной частицы обозначается буквой «u».

Природные типы магнитной индукции следующие.

  • Fмач– наибольшая сила, воздействующая на проводник.
  • L – его длина.
  • I – сила тока заряженных частиц в металле.

Единицей индуктивности в международной системе СИ является «Тесла», сокращенно обозначаемый как «ТЛ» на русском языке и как «Т» в международной системе. Он назван в честь сербского ученого Н. Теслы. В старой метрической системе СГС индуктивные единицы назывались по немецкому природному «гаусс». GS между ученым и Г. Русский язык для международной версии.

Магнитное поле

В физике этот термин относится к силовому полю, оказывающему особое воздействие на заряженные частицы и другие тела с определенным магнитным моментом. Это явление не зависит от того, является ли частица неподвижной или движущейся. Помимо вектора магнитной индукции, дополнительной характеристикой поля является векторный потенциал. Это альтернативный, но, с точки зрения природы, тесно связанный с магнитной индукцией способ.

В случае полей, действующих в абсолютном промежутке, основной характеристикой обычно является не индукция, а ее сила, символизируемая H. Однако такая замена поля на магнитное поле Земли уже не имеет большого смысла. Поэтому в реальных экспериментальных измерениях и наблюдениях вектор магнитной индукции по-прежнему используется в качестве основной характеристики магнитного поля.

Действительно, магнитное поле можно определить как особый предмет. Это взаимодействие между определенными заряженными элементарными частицами, движущимися с определенной скоростью.

Не путайте магнетизм и электромагнитную индукцию. Электромагнитная индукция относится к закономерности, введенной англичанином М. Фарадеем. Суть закона заключается в возникновении электромагнитного потенциального поля под воздействием переменного тока, протекающего в замкнутом проводниковом контуре. В цепи создается определенная движущая сила, вызывающая индукционный ток. Помимо электрического поля, магнитное поле является одной из двух частей электромагнитного поля.

Теория постоянных магнитов, вызывающих индукцию, была разработана французским естествоиспытателем A-M. Ампер. Ампера, от имени которого впоследствии была названа электростанция. Он первым обнаружил, что движение электронов вокруг центра индивида в конечном итоге приводит к возникновению небольшого или фундаментального магнитного поля. Он также открыл свойства металлических проводников сохранять свои магнитные свойства. После прекращения приложения магнитного поля на некоторое время.

Магнитная индукция (символ B) является основной характеристикой магнитного поля (величина вектора) и определяет работу выхода движущегося заряда (тока) из магнитного поля и направлена перпендикулярно направлению скорости.

B и H магнитная индукция

Магнитная индукция измеряется в Теслах (СИ), где ТЛ эквивалентна n S / Cl m. H — в ньютонах, а единица силы — второе время cl. СГС использует гаусс для той же цели (hs =√g / s√cm). Здесь g магнитная индукция измеряется в амперах на метр (СИ) или ЭРСТЕАДАХ (ЭГС). В русской литературе h называют напряженностью поля.

Блок Тесла был введен в 1960 году Международной конференцией по весам и мерам в честь покойного Николы Тесла. На самом деле, с самого начала Si. Как жили ученые до этого? СГС уже существовала, когда в 1948 году возникла идея введения СИ. Корни последнего были установлены Карлом Фридрихом Гауссом в 1832 году. Карл Фридрих Гаусс искал единое основание в области физики, чтобы облегчить соотнесение различных законов. Ученый установил три основные единицы измерения: миллиметр, миллиграмм и секунда.

Гаус умер вскоре после введения понятия магнитной индукции и деления на В и О. В 1874 году Джеймс Максвелл и лорд Кельвин добавили к этому списку новую единицу измерения. Магнитная индукция была названа в честь ее основателя, и в то же время система была названа в его честь (ранее она называлась системой Гаусса). Что касается СИ, то Тесла может быть выражена по-разному, либо через основные единицы, либо через производные единицы. Weber, указана за квадратный метр.

Отталкивание катушки тока.

В вакууме два типа индукции (H и B) связаны через константы. Чтобы отличить их друг от друга, OR называют вектором напряженности магнитного поля. Очевидно, что смысл не сильно отличается от B.

  1. μ – магнитная проницаемость среды.
  2. μ0 – магнитная постоянная (проницаемость вакуума). В системе СГС равна 1, в вакууме В и Н одинаковы. В СИ составляет 1,257 микроньютона на квадратный ампер.

Константы введены намеренно, чтобы связать свойства магнитных полей H и B. Кстати, существует много версий, почему лорд Кельвин назвал векторы именно так (буквы H и B). Если интересно, рекомендуется ознакомиться с терминами относительная магнитная проницаемость (причина постоянного отношения M0 к абсолютному m) и магнитное сомнение (относительная магнитная проницаемость, увеличенная на 1). Это помогает лучше понять типы литературы, которые по-разному связаны с В.

Можно найти множество законов и типов магнитной индукции. Это показывает, насколько теоретически важен этот параметр. Автор не знает, использовал ли Никола Тесла подобную величину при разработке своего многофазного индукционного двигателя, но не зря же эта величина названа в честь великого ученого!

Измерения ЭМП, критерии воздействия и дозиметрия – оценка программы экологического мониторинга системы сверхнизкочастотной связи ВМС США

Изучение воздействия электрических и магнитных полей (ЭМП) на организмы включает точную оценку воздействия этих полей и то, что организм получает в результате воздействия. Воздействие – это мера напряженности поля электрического или магнитного поля непосредственно вне организма в течение определенного периода времени. Доза является мерой наведенной напряженности поля внутри организма за определенный период. В первом разделе этой главы описывается, как IITRI охарактеризовал ЭМП вблизи передающих устройств. В последующих разделах обсуждаются проблемы, связанные с оценкой доз и учетом возможных эффектов, связанных с модуляцией сигнала.

Термин «ЭМП» применяется к переменному полю, создаваемому движущимися заряженными частицами. ЭМП характеризуются длиной волны (выраженной в метрах) и частотой (выраженной в герцах). Длина волны поля, умноженная на его частоту, равна скорость распространения. Полный диапазон частот ЭМП описывается как электромагнитный спектр. Обозначение «чрезвычайно низкочастотный» (ELF) обычно зарезервировано для частот в диапазоне от 3 Гц до 300 Гц. Большинство оборудования, используемого для генерации , передача и распределение электроэнергии в Соединенных Штатах генерирует ЭМП с частотой 60 Гц. Система связи ELF ВМФ использует принцип частотной модуляции, называемый минимальной манипуляцией. В этом типе модуляции частота сдвигается между 72 Гц и 80 Гц (с центром 76 Гц) в зависимости от того, должен ли передаваться на подводную лодку код «единица» или «ноль» (Запотоский и др. 19).96). Интенсивность электрических полей выражается в вольтах на метр (В/м), а магнитных полей выражается в миллигауссах (мГс). Дополнительная информация предоставлена ​​NIOSH, NIEHS и DOE (1996).

Характеристика электрических и магнитных полей

Для характеристики электрических и магнитных полей вблизи объектов экологического мониторинга IITRI измерил пространственные и временные характеристики следующих полей:

  • Магнитное поле в воздухе и на земле, создаваемое электрический ток в антенне и клеммах заземления.

  • Электрическое поле в земле, представляющее собой сумму полей, индуцированных магнитным полем, и тока, протекающего от заглубленных заземляющих клемм.

  • Электрическое поле в воздухе, возникающее из-за разности электрических потенциалов между антеннами и землей или создаваемое как побочный продукт электрического поля в земле.

  • Статическое геомагнитное поле Земли.

IITRI предоставил следующие размеры ЭМП вблизи передающих устройств (см. используемые приборы):

ТАБЛИЦА 2-1

Инструменты, используемые IITRI для измерения электромагнитных полей.

1.

Окружающая среда 60 Гц результирующая 1 ЭМП над землей.

2.

Немодулированные результирующие ЭМП 76 Гц над землей.

3.

Модулированные результирующие ЭМП 76 Гц над землей.

4.

Среднеквадратические значения гармоник ЭМП 60 Гц и 76 Гц над землей.

5.

Разность потенциалов земли в двух ортогональных направлениях и, на основании этого, результирующее электрическое поле в земле.

Возможность измерения магнитных полей низкого уровня зависит, среди прочего, от чувствительности прибора, используемого для анализа напряжения зонда магнитного поля. Согласно IITRI, оборудование для измерения магнитного поля было откалибровано путем измерения выходного напряжения зонда магнитного поля, помещенного в магнитное поле силой 100 мГс (Haradem et al. 19).94). В нем утверждается, что эта калибровка действительна при меньших уровнях поля на основании того факта, что датчик, построенный исключительно из пассивных компонентов, как известно, имеет выходной сигнал, линейный по отношению к напряженности поля. Наименьшая полномасштабная чувствительность прибора IITRI (Hewlett-Packard 3581A) составляла 0,1 мкВ, что соответствует уровню магнитного поля около 0,2 мГс. Ожидается, что измерения, намного более низкие, чем это (около 0,02 мГ или ниже), не будут очень точными. К счастью, наиболее важные сообщаемые уровни, использованные при создании участков обработки и контроля, а также при анализе экологических данных, были намного выше этих уровней и, как ожидается, будут точными представлениями магнитных полей. Однако ожидается, что сообщаемые уровни поля, такие как 0,0002 мГс, не будут точными.

Чтобы исключить возможность загрязнения экологических исследований гармониками или взаимодействиями между частотами линии электропередач и частотами антенны СНЧ, был измерен спектр зависимости напряженности поля от частоты. Все нежелательные сигналы оказались как минимум на 30 дБ ниже уровня частот антенны СНЧ, и поэтому было сочтено, что они не мешают экологическим исследованиям. Сообщалось, что среднеквадратические значения гармоник электрических и магнитных полей частотой 60 Гц и 76 Гц над землей либо ниже уровней обнаружения, либо «настолько низки, что их нельзя считать помехой». Спектры, измеренные IITRI на антенных терминалах с выключенным передатчиком и с включенным передатчиком, дают данные об окружающих полях 60 Гц и полях 76 Гц (J.R. Gauger, IITRI, письмо в Управление проекта систем связи ВМС США, 23 декабря 1985). Хотя приведенные спектральные данные относятся к наблюдениям за один день (11 декабря 1985 г. ), они подтверждают приведенное выше утверждение. 2 Признаки естественных резонансов Шумана в земной атмосфере и измеренное поведение гармоник также подтверждают качество наблюдений. Несколько явных ошибок в метках на спектрах согласуются с проблемами натурных наблюдений и легко устраняются. Однако таких ошибок было относительно мало.

Критерии воздействия для выбора места

Любой источник электрических и магнитных полей (например, передающие устройства сверхнизкой частоты и антенны) создает поля практически везде. По мере удаления от источника напряженность поля становится ниже либо из-за расстояния, либо из-за ослабления из-за препятствий (в случае электрических полей). Однако тогда человек будет двигаться к другим источникам, и генерируемые ими ЭМП будут увеличиваться по интенсивности по мере приближения. Поэтому невозможно выбрать место управления, где нет воздействия ЭМП СНЧ, генерируемого передающими средствами и антеннами ВМФ. Можно только выбрать места, которые имеют разные уровни воздействия ЭМП, генерируемого антеннами, и других источников, таких как линии электропередач.

IITRI помог исследователям выбрать участки для программы экологического мониторинга, определив, относятся ли они к категории обработки или контроля. Конкретные критерии, используемые для определения того, был ли участок лечебным или контрольным, были следующими:

, где T(76 Гц) – воздействие на лечебный участок из-за системы связи ELF, T(60 Гц) – облучение на лечебном участке из-за от линий электропередач, C(76 Гц) — воздействие на контрольную точку из-за системы связи ELF, а C(60 Гц) — воздействие на контрольную точку из-за линий электропередач.

Другими словами, интенсивность ЭМП 76 Гц в месте лечения должна была быть в 10 раз больше, чем интенсивность ЭМП 76 Гц в контрольной зоне. Кроме того, как на лечебных, так и на контрольных участках интенсивность ЭМП 76 Гц от антенн должна была в 10 раз превышать интенсивность ЭМП 60 Гц от близлежащих линий электропередач. Наконец, отношение интенсивностей полей частотой 60 Гц в пункте обработки и контроля должно быть в пределах от 0,1 до 10. Эти критерии применялись к ЭМП в воздухе и на земле при работе на полной мощности соответствующей передающей антенны. . Однако не было априорных доказательств того, что уменьшение воздействия на одну десятую должно привести к уменьшению на одну десятую (или меньше) эффекта, который можно было бы наблюдать при полном воздействии.

Изменение интенсивности поля в зависимости от расстояния означает, что каждое место подвергается воздействию пространственного градиента интенсивности, а не равномерной интенсивности по всему участку. Поэтому участки были классифицированы в соответствии с ежегодными измерениями, проводимыми в одной и той же точке каждый год.

Чтобы изолировать эффекты ЭМП КНЧ, парные участки обработки и контроля должны были быть как можно более похожими в отношении экологических переменных, включая почвы, листву, обилие видов и температуру, в зависимости от направленности исследования. Например, для исследования водно-болотных угодий в качестве участков обработки и контроля требовались аналогичные болота, тогда как для исследования насекомых-опылителей требовались участки с одинаковым обилием цветов. Несколько исследовательских групп столкнулись с трудностями при определении пар участков, отвечающих как критериям воздействия, так и экологическим критериям, как описано в главе 39.0007

Данные о воздействии, предоставленные исследователям

IITRI предоставил исследователям данные о магнитных полях и электрических полях в воздухе, а также электрических полях в земле. IITRI предоставил группам экологического мониторинга обширные данные о конкретных измерениях ЭМП КНЧ на каждом участке. Цель этих измерений состояла в том, чтобы позволить группам мониторинга определить индикаторы воздействия для различных частей участков обработки и попытаться связать индикаторы с соответствующими показателями экологического воздействия.

IITRI также предоставил данные о времени включения и выключения передатчика группам экологического мониторинга. Эти данные могут быть использованы для определения того, действительно ли место, охарактеризованное как место лечения, подвергалось воздействию ЭМП СНЧ от антенны в какое-либо конкретное время. Это важно, потому что передатчик не был включен постоянно, а место лечения подвергается воздействию только тогда, когда передатчик включен.

Использование формул для прогнозирования электрических и магнитных полей

Электрические и магнитные поля могут быть охарактеризованы либо физическими измерениями, либо теоретически, хотя оба метода сопряжены с трудностями. Физическое измерение напряженности поля может быть затруднено из-за ограничений измерительного оборудования и градиентов напряженности поля, вызванных расстоянием от источника и изменениями рельефа местности. Такие поля часто слишком сложны, чтобы их можно было адекватно охарактеризовать с помощью простых формул. Магнитное поле частотой 76 Гц было относительно устойчивым и могло быть хорошо охарактеризовано в пространстве с помощью простых формул. Однако для характеристики электрических полей были необходимы физические измерения. Пространственная зависимость электрических полей в земле не могла быть предсказана с помощью простых формул из-за пространственной изменчивости проводимости земли. Кроме того, имелись (обычно скромные) временные колебания из-за ежедневных и годовых изменений электропроводности Земли. Необходимо было провести тщательные исследования пространственных и временных изменений электрических полей Земли. Электрическое поле в воздухе имеет характеристики, которые находятся между характеристиками магнитного поля и электрического поля в земле. На открытой местности с ровным рельефом это поле хорошо характеризуется простыми формулами. Однако при наличии препятствий, таких как деревья, электрические поля в воздухе рассчитать сложно, и измерения дают только моментальные снимки, поскольку поле переменное. Например, ветер, движущийся сквозь листву, вызывает изменчивость электрических полей в воздухе.

IITRI предоставил несколько формул, которые можно использовать для расчета ЭМП сверхнизких частот вблизи антенн. Не все ограничения этих формул были указаны в отчетах, полученных комитетом. Авторы правильно указали, что формулы подходят только для точек поля на поверхности земли и вблизи антенн. Однако есть и другие ограничения. Во-первых, все формулы ограничены квазистатическим диапазоном частот, поэтому все соответствующие расстояния должны быть существенно меньше длины волны в земле. Во-вторых, каждый предполагает идеальную плоскую однородную землю. Некоторые из этих предположений верны, другие нет и исключают использование формул. Например, разумно рассчитать магнитные поля (либо в воздухе, либо в земле) с помощью простого закона Био-Савара, предоставленного IITRI, если известен ток антенны (какой он есть). Но обычно неверно предполагать, что проводимость земли однородна или не зависит от времени, для расчетов электрических полей в земле. Если необходимо учитывать изменения электропроводности в зависимости от глубины или горизонтального положения, формула, приведенная IITRI, недействительна. Фактически было обнаружено, что электропроводность существенно различалась на некоторых исследуемых участках и зависела от условий окружающей среды и, следовательно, от времени года. Вот почему следует быть очень осторожным в использовании этой формулы и почему необходимы дополнительные измерения электрических полей в земле. Осторожность, требуемая при использовании формулы для электрических полей в воздухе, находится между этими крайностями. Для однородной ровной местности без препятствий приведенные формулы удовлетворительны (при условии, что напряжение антенны известно). Однако в роще поля сильно искажены, и формула, предложенная IITRI, бесполезна.

Из формул, предоставленных IITRI, только формула магнитного поля широко использовалась исследователями. Эта формула использовалась только при попытке интерполировать магнитные поля в точках в пределах исследуемых участков. Для изучения горной флоры были проведены измерения магнитного поля частотой 76 Гц в нескольких точках вблизи висконсинской антенны. сравнивает измеренные с расчетными полями для высоты антенны 13,7 м и тока антенны 150 А. Измеренные и расчетные значения согласуются достаточно хорошо.

ТАБЛИЦА 2-2

Сравнение измеренных и рассчитанных магнитных полей.

К сожалению, IITRI не сообщил об измерениях напряжения между антенной и землей, которые можно было бы использовать для подтверждения результатов измерений электрических полей в воздухе. Однако это напряжение можно оценить следующим образом. Согласно Диллу (1984), одним из критериев проектирования клемм заземления было достижение максимального общего сопротивления заземления 6 Ом для обоих заземлений. На основании этого числа и предположения, что сопротивление антенны намного меньше сопротивления земли, напряжение между антенной и землей при полном токе (150 А) составляет 900 В. Согласно формуле, предоставленной IITRI, электрическое поле в воздухе непосредственно под антенной будет около 25 В/м. Измеренное электрическое поле под антенной для исследований мелких млекопитающих и гнездящихся птиц составляло 10-40 В/м. Из этого результата и теоретических оценок можно сделать вывод, что измерения находятся в разумном согласии.

Дозиметрия

Как указано выше, величины ЭМП, относящиеся к взаимодействию КНЧ с биологическими системами, представляют собой воздействие (напряженность поля сразу вне организма за период времени) и дозу (индуцированное поле внутри организмов за период времени). Последние величины могут быть выражены через напряженность индуцированного электрического поля, напряженность магнитного поля и индуцированный ток или плотность тока. Дозиметрия включает оценку величины и распределения индуцированных полей и токов в биологических организмах, подвергающихся воздействию ЭМП КНЧ. Индуцированные поля и токи не только являются функциями наложенных извне ЭМП, но и определяются свойствами ЭМП и геометрией подвергаемого воздействию организма и любых близлежащих объектов. Не следует ожидать, что такие индуцированные поля и токи на КНЧ будут составлять дозу.

Дозиметрические измерения индуцированных полей и токов не входили в план исследования. Тем не менее, чтобы получить некоторые данные об относительной силе индуцированных электрических полей в различных биотах, подвергшихся воздействию ЭМП частотой 76 Гц, комитет провел ряд анализов с использованием простых моделей, которые служат в качестве показателя индуцированных полей внутри организма. Среда воздействия КНЧ-ЭМП была охарактеризована на контрольных и лечебных участках посредством периодических обследований. Эта среда включала поля с частотой 76 Гц, создаваемые системой связи ELF, поля с частотой 60 Гц от линий электропередач и магнитное поле Земли. Поскольку длина волны 76 Гц намного длиннее самого длинного измерения организма, квазистатическая теория поля может быть надлежащим образом применена для расчета индуцированного электрического поля внутри тела организма (Майклсон и Лин 19).87).

Вкратце, результаты расчетов показывают, что индуцированные электрические поля у насекомых, птиц и мелких позвоночных довольно низки при воздействии внешних электрических полей до 5000 мВ/м и магнитных полей до 50 мГс. Напротив, электрические поля, создаваемые в лиственных насаждениях теми же ЭМП, могут быть значительными. Расчеты, основанные на этих простых моделях, показывают, что поле, создаваемое вертикально ориентированным электрическим полем в 25-метровом дереве, может достигать 5000 мВ/м, а поле, создаваемое горизонтально ориентированным магнитным полем, — до 29 мВ/м.0,8 мВ/м. Сила приложенного или падающего электрического поля будет уменьшаться по мере удаления от провода антенны и из-за экранирования. Однако напряженность магнитного поля будет ослабевать вдали от антенного провода только на расстоянии. Поэтому на больших расстояниях от антенны поле, индуцированное в древостоях горизонтальным магнитным полем, может стать доминирующим фактором в результирующей дозе. (Более подробную информацию см. в Приложении B.)

Исследователей, участвующих в программе экологического мониторинга, не просили оценить дозы, полученные биотой от передающих антенн КНЧ ВМФ. Только исследователи, занимавшиеся изучением горной флоры, пытались это сделать. Поскольку по дозиметрии не было достаточной информации, комитет решил, что невозможно экстраполировать результаты программы мониторинга на другие ситуации, которые могут быть сопоставимы с условиями облучения.

Различия в действии между немодулированными сигналами 60 Гц и модулированными сигналами 76 Гц

Как упоминалось ранее, электрические и магнитные поля СНЧ, генерируемые антеннами системы связи, модулируются по частоте между 72 и 80 Гц (с преобладающей частотой 76 Гц). ), в отличие от ЭМП линий электропередач, которые не модулируются на частоте 60 Гц. К сожалению, мало информации о различиях между эффектами модулированных и немодулированных частот. Большинство исследований, предпринятых за последние 25 лет для понимания биологических эффектов низкочастотных ЭМП, были сосредоточены на воздействии немодулированных ЭМП на частотах сети 50-60 Гц (см., например, Anderson 19).90; ОРАУ 1992 г.; Тенфорд 1996; ОТА 1989 г.; NRC 1997). Было проведено мало исследований влияния модулированных сигналов частотой 76 Гц, создаваемых системой связи ELF.

Выводы относительно измерений ЭМП

IITRI проделал хорошую общую работу по характеристике электрических и магнитных полей сверхнизких частот вблизи мест обработки и контроля. В тех случаях, когда становилось очевидным, что требуется дополнительная информация, IITRI реагировал и проводил дополнительные измерения. Конкретные выводы таковы:

  • Несмотря на то, что были некоторые незначительные вопросы по конструкции прибора, оказалось, что связанные с этим ошибки были небольшими и не привели к изменению выводов IITRI относительно данных измерений.

  • Хорошо охарактеризованы пространственные и временные вариации магнитных полей над землей вблизи участков обработки и контроля.

  • Хорошо охарактеризованы пространственные и временные вариации электрических полей над землей на открытых площадках вблизи очистных и контрольных участков.

    В защищенных зонах, таких как деревья, необходимы более обширные измерения. По запросу их предоставили.

  • Электрические поля в земле зависят от локальной проводимости земли, поэтому для их характеристики требуются более тщательные измерения. По запросу IITRI предоставил инженерную поддержку для этих измерений.

  • Электрические поля Земли исследованы в свете ежегодных изменений электрических характеристик Земли. Большинство вариаций были скромными, но в этих полях происходили ежедневные и годовые изменения.

Сноски

1

“Результат” определяется следующим образом. Среднеквадратичные (среднеквадратические) величины трех прямоугольных составляющих поля определяются либо измерением, либо вычислением. синусоидально во времени, среднеквадратичное значение каждой составляющей представляет собой величину от нуля до пика, деленную на квадратный корень из 2. Результатом является квадратный корень из суммы квадратов этих трех среднеквадратичных значений

2

Сигнал 60 Гц линии электропередач на 30 дБ ниже сигнала передатчика 76 Гц; а самая сильная гармоника частоты линии электропередач (300 Гц) не менее чем на 60 дБ ниже сигнала передатчика. Спектры показывают гармоники частоты окружающего электроснабжения до 17-й гармоники и частоты передатчика до 11-й гармоники. Гармоники передатчика ниже основной на 35 дБ при 216 Гц и 240 Гц, на 50 дБ при 144 Гц и 160 Гц и на 55 дБ при 360 Гц и 400 Гц.

Измерения ЭМП, критерии воздействия и дозиметрия — оценка программы экологического мониторинга системы сверхнизкочастотной связи ВМС США

Изучение воздействия электрических и магнитных полей (ЭМП) на организмы включает точную оценку воздействия этих полей и того, что организм получает в результате воздействия. Экспозиция – это мера напряженности поля электрического или магнитного поля непосредственно снаружи организма за определенный период. Доза является мерой напряженности поля , индуцированного внутри организма в течение определенного периода времени. В первом разделе этой главы описывается, как IITRI охарактеризовал ЭМП вблизи передающих устройств. В последующих разделах обсуждаются проблемы, связанные с оценкой доз и учетом возможных эффектов, связанных с модуляцией сигнала.

Термин «ЭМП» применяется к переменному полю, создаваемому движущимися заряженными частицами. ЭМП характеризуются длиной волны (выраженной в метрах) и частотой (выраженной в герцах). Длина волны поля, умноженная на его частоту, равна скорость распространения. Полный диапазон частот ЭМП описывается как электромагнитный спектр. Обозначение «чрезвычайно низкочастотный» (ELF) обычно зарезервировано для частот в диапазоне от 3 Гц до 300 Гц. Большинство оборудования, используемого для генерации , передача и распределение электроэнергии в Соединенных Штатах генерирует ЭМП с частотой 60 Гц. Система связи ELF ВМФ использует принцип частотной модуляции, называемый минимальной манипуляцией. В этом типе модуляции частота сдвигается между 72 Гц и 80 Гц (с центром 76 Гц) в зависимости от того, должен ли передаваться на подводную лодку код «единица» или «ноль» (Запотоский и др. 19).96). Интенсивность электрических полей выражается в вольтах на метр (В/м), а магнитных полей выражается в миллигауссах (мГс). Дополнительная информация предоставлена ​​NIOSH, NIEHS и DOE (1996).

Характеристика электрических и магнитных полей

Для характеристики электрических и магнитных полей вблизи объектов экологического мониторинга IITRI измерил пространственные и временные характеристики следующих полей:

  • Магнитное поле в воздухе и на земле, создаваемое электрический ток в антенне и клеммах заземления.

  • Электрическое поле в земле, представляющее собой сумму полей, индуцированных магнитным полем, и тока, протекающего от заглубленных заземляющих клемм.

  • Электрическое поле в воздухе, возникающее из-за разности электрических потенциалов между антеннами и землей или создаваемое как побочный продукт электрического поля в земле.

  • Статическое геомагнитное поле Земли.

IITRI предоставил следующие размеры ЭМП вблизи передающих устройств (см. используемые приборы):

ТАБЛИЦА 2-1

Инструменты, используемые IITRI для измерения электромагнитных полей.

1.

Окружающая среда 60 Гц результирующая 1 ЭМП над землей.

2.

Немодулированные результирующие ЭМП 76 Гц над землей.

3.

Модулированные результирующие ЭМП 76 Гц над землей.

4.

Среднеквадратические значения гармоник ЭМП 60 Гц и 76 Гц над землей.

5.

Разность потенциалов земли в двух ортогональных направлениях и, на основании этого, результирующее электрическое поле в земле.

Возможность измерения магнитных полей низкого уровня зависит, среди прочего, от чувствительности прибора, используемого для анализа напряжения зонда магнитного поля. Согласно IITRI, оборудование для измерения магнитного поля было откалибровано путем измерения выходного напряжения зонда магнитного поля, помещенного в магнитное поле силой 100 мГс (Haradem et al. 19).94). В нем утверждается, что эта калибровка действительна при меньших уровнях поля на основании того факта, что датчик, построенный исключительно из пассивных компонентов, как известно, имеет выходной сигнал, линейный по отношению к напряженности поля. Наименьшая полномасштабная чувствительность прибора IITRI (Hewlett-Packard 3581A) составляла 0,1 мкВ, что соответствует уровню магнитного поля около 0,2 мГс. Ожидается, что измерения, намного более низкие, чем это (около 0,02 мГ или ниже), не будут очень точными. К счастью, наиболее важные сообщаемые уровни, использованные при создании участков обработки и контроля, а также при анализе экологических данных, были намного выше этих уровней и, как ожидается, будут точными представлениями магнитных полей. Однако ожидается, что сообщаемые уровни поля, такие как 0,0002 мГс, не будут точными.

Чтобы исключить возможность загрязнения экологических исследований гармониками или взаимодействиями между частотами линии электропередач и частотами антенны СНЧ, был измерен спектр зависимости напряженности поля от частоты. Все нежелательные сигналы оказались как минимум на 30 дБ ниже уровня частот антенны СНЧ, и поэтому было сочтено, что они не мешают экологическим исследованиям. Сообщалось, что среднеквадратические значения гармоник электрических и магнитных полей частотой 60 Гц и 76 Гц над землей либо ниже уровней обнаружения, либо «настолько низки, что их нельзя считать помехой». Спектры, измеренные IITRI на антенных терминалах с выключенным передатчиком и с включенным передатчиком, дают данные об окружающих полях 60 Гц и полях 76 Гц (J.R. Gauger, IITRI, письмо в Управление проекта систем связи ВМС США, 23 декабря 1985). Хотя приведенные спектральные данные относятся к наблюдениям за один день (11 декабря 1985 г. ), они подтверждают приведенное выше утверждение. 2 Признаки естественных резонансов Шумана в земной атмосфере и измеренное поведение гармоник также подтверждают качество наблюдений. Несколько явных ошибок в метках на спектрах согласуются с проблемами натурных наблюдений и легко устраняются. Однако таких ошибок было относительно мало.

Критерии воздействия для выбора места

Любой источник электрических и магнитных полей (например, передающие устройства сверхнизкой частоты и антенны) создает поля практически везде. По мере удаления от источника напряженность поля становится ниже либо из-за расстояния, либо из-за ослабления из-за препятствий (в случае электрических полей). Однако тогда человек будет двигаться к другим источникам, и генерируемые ими ЭМП будут увеличиваться по интенсивности по мере приближения. Поэтому невозможно выбрать место управления, где нет воздействия ЭМП СНЧ, генерируемого передающими средствами и антеннами ВМФ. Можно только выбрать места, которые имеют разные уровни воздействия ЭМП, генерируемого антеннами, и других источников, таких как линии электропередач.

IITRI помог исследователям выбрать участки для программы экологического мониторинга, определив, относятся ли они к категории обработки или контроля. Конкретные критерии, используемые для определения того, был ли участок лечебным или контрольным, были следующими:

, где T(76 Гц) – воздействие на лечебный участок из-за системы связи ELF, T(60 Гц) – облучение на лечебном участке из-за от линий электропередач, C(76 Гц) — воздействие на контрольную точку из-за системы связи ELF, а C(60 Гц) — воздействие на контрольную точку из-за линий электропередач.

Другими словами, интенсивность ЭМП 76 Гц в месте лечения должна была быть в 10 раз больше, чем интенсивность ЭМП 76 Гц в контрольной зоне. Кроме того, как на лечебных, так и на контрольных участках интенсивность ЭМП 76 Гц от антенн должна была в 10 раз превышать интенсивность ЭМП 60 Гц от близлежащих линий электропередач. Наконец, отношение интенсивностей полей частотой 60 Гц в пункте обработки и контроля должно быть в пределах от 0,1 до 10. Эти критерии применялись к ЭМП в воздухе и на земле при работе на полной мощности соответствующей передающей антенны. . Однако не было априорных доказательств того, что уменьшение воздействия на одну десятую должно привести к уменьшению на одну десятую (или меньше) эффекта, который можно было бы наблюдать при полном воздействии.

Изменение интенсивности поля в зависимости от расстояния означает, что каждое место подвергается воздействию пространственного градиента интенсивности, а не равномерной интенсивности по всему участку. Поэтому участки были классифицированы в соответствии с ежегодными измерениями, проводимыми в одной и той же точке каждый год.

Чтобы изолировать эффекты ЭМП КНЧ, парные участки обработки и контроля должны были быть как можно более похожими в отношении экологических переменных, включая почвы, листву, обилие видов и температуру, в зависимости от направленности исследования. Например, для исследования водно-болотных угодий в качестве участков обработки и контроля требовались аналогичные болота, тогда как для исследования насекомых-опылителей требовались участки с одинаковым обилием цветов. Несколько исследовательских групп столкнулись с трудностями при определении пар участков, отвечающих как критериям воздействия, так и экологическим критериям, как описано в главе 39.0007

Данные о воздействии, предоставленные исследователям

IITRI предоставил исследователям данные о магнитных полях и электрических полях в воздухе, а также электрических полях в земле. IITRI предоставил группам экологического мониторинга обширные данные о конкретных измерениях ЭМП КНЧ на каждом участке. Цель этих измерений состояла в том, чтобы позволить группам мониторинга определить индикаторы воздействия для различных частей участков обработки и попытаться связать индикаторы с соответствующими показателями экологического воздействия.

IITRI также предоставил данные о времени включения и выключения передатчика группам экологического мониторинга. Эти данные могут быть использованы для определения того, действительно ли место, охарактеризованное как место лечения, подвергалось воздействию ЭМП СНЧ от антенны в какое-либо конкретное время. Это важно, потому что передатчик не был включен постоянно, а место лечения подвергается воздействию только тогда, когда передатчик включен.

Использование формул для прогнозирования электрических и магнитных полей

Электрические и магнитные поля могут быть охарактеризованы либо физическими измерениями, либо теоретически, хотя оба метода сопряжены с трудностями. Физическое измерение напряженности поля может быть затруднено из-за ограничений измерительного оборудования и градиентов напряженности поля, вызванных расстоянием от источника и изменениями рельефа местности. Такие поля часто слишком сложны, чтобы их можно было адекватно охарактеризовать с помощью простых формул. Магнитное поле частотой 76 Гц было относительно устойчивым и могло быть хорошо охарактеризовано в пространстве с помощью простых формул. Однако для характеристики электрических полей были необходимы физические измерения. Пространственная зависимость электрических полей в земле не могла быть предсказана с помощью простых формул из-за пространственной изменчивости проводимости земли. Кроме того, имелись (обычно скромные) временные колебания из-за ежедневных и годовых изменений электропроводности Земли. Необходимо было провести тщательные исследования пространственных и временных изменений электрических полей Земли. Электрическое поле в воздухе имеет характеристики, которые находятся между характеристиками магнитного поля и электрического поля в земле. На открытой местности с ровным рельефом это поле хорошо характеризуется простыми формулами. Однако при наличии препятствий, таких как деревья, электрические поля в воздухе рассчитать сложно, и измерения дают только моментальные снимки, поскольку поле переменное. Например, ветер, движущийся сквозь листву, вызывает изменчивость электрических полей в воздухе.

IITRI предоставил несколько формул, которые можно использовать для расчета ЭМП сверхнизких частот вблизи антенн. Не все ограничения этих формул были указаны в отчетах, полученных комитетом. Авторы правильно указали, что формулы подходят только для точек поля на поверхности земли и вблизи антенн. Однако есть и другие ограничения. Во-первых, все формулы ограничены квазистатическим диапазоном частот, поэтому все соответствующие расстояния должны быть существенно меньше длины волны в земле. Во-вторых, каждый предполагает идеальную плоскую однородную землю. Некоторые из этих предположений верны, другие нет и исключают использование формул. Например, разумно рассчитать магнитные поля (либо в воздухе, либо в земле) с помощью простого закона Био-Савара, предоставленного IITRI, если известен ток антенны (какой он есть). Но обычно неверно предполагать, что проводимость земли однородна или не зависит от времени, для расчетов электрических полей в земле. Если необходимо учитывать изменения электропроводности в зависимости от глубины или горизонтального положения, формула, приведенная IITRI, недействительна. Фактически было обнаружено, что электропроводность существенно различалась на некоторых исследуемых участках и зависела от условий окружающей среды и, следовательно, от времени года. Вот почему следует быть очень осторожным в использовании этой формулы и почему необходимы дополнительные измерения электрических полей в земле. Осторожность, требуемая при использовании формулы для электрических полей в воздухе, находится между этими крайностями. Для однородной ровной местности без препятствий приведенные формулы удовлетворительны (при условии, что напряжение антенны известно). Однако в роще поля сильно искажены, и формула, предложенная IITRI, бесполезна.

Из формул, предоставленных IITRI, только формула магнитного поля широко использовалась исследователями. Эта формула использовалась только при попытке интерполировать магнитные поля в точках в пределах исследуемых участков. Для изучения горной флоры были проведены измерения магнитного поля частотой 76 Гц в нескольких точках вблизи висконсинской антенны. сравнивает измеренные с расчетными полями для высоты антенны 13,7 м и тока антенны 150 А. Измеренные и расчетные значения согласуются достаточно хорошо.

ТАБЛИЦА 2-2

Сравнение измеренных и рассчитанных магнитных полей.

К сожалению, IITRI не сообщил об измерениях напряжения между антенной и землей, которые можно было бы использовать для подтверждения результатов измерений электрических полей в воздухе. Однако это напряжение можно оценить следующим образом. Согласно Диллу (1984), одним из критериев проектирования клемм заземления было достижение максимального общего сопротивления заземления 6 Ом для обоих заземлений. На основании этого числа и предположения, что сопротивление антенны намного меньше сопротивления земли, напряжение между антенной и землей при полном токе (150 А) составляет 900 В. Согласно формуле, предоставленной IITRI, электрическое поле в воздухе непосредственно под антенной будет около 25 В/м. Измеренное электрическое поле под антенной для исследований мелких млекопитающих и гнездящихся птиц составляло 10-40 В/м. Из этого результата и теоретических оценок можно сделать вывод, что измерения находятся в разумном согласии.

Дозиметрия

Как указано выше, величины ЭМП, относящиеся к взаимодействию КНЧ с биологическими системами, представляют собой воздействие (напряженность поля сразу вне организма за период времени) и дозу (индуцированное поле внутри организмов за период времени). Последние величины могут быть выражены через напряженность индуцированного электрического поля, напряженность магнитного поля и индуцированный ток или плотность тока. Дозиметрия включает оценку величины и распределения индуцированных полей и токов в биологических организмах, подвергающихся воздействию ЭМП КНЧ. Индуцированные поля и токи не только являются функциями наложенных извне ЭМП, но и определяются свойствами ЭМП и геометрией подвергаемого воздействию организма и любых близлежащих объектов. Не следует ожидать, что такие индуцированные поля и токи на КНЧ будут составлять дозу.

Дозиметрические измерения индуцированных полей и токов не входили в план исследования. Тем не менее, чтобы получить некоторые данные об относительной силе индуцированных электрических полей в различных биотах, подвергшихся воздействию ЭМП частотой 76 Гц, комитет провел ряд анализов с использованием простых моделей, которые служат в качестве показателя индуцированных полей внутри организма. Среда воздействия КНЧ-ЭМП была охарактеризована на контрольных и лечебных участках посредством периодических обследований. Эта среда включала поля с частотой 76 Гц, создаваемые системой связи ELF, поля с частотой 60 Гц от линий электропередач и магнитное поле Земли. Поскольку длина волны 76 Гц намного длиннее самого длинного измерения организма, квазистатическая теория поля может быть надлежащим образом применена для расчета индуцированного электрического поля внутри тела организма (Майклсон и Лин 19).87).

Вкратце, результаты расчетов показывают, что индуцированные электрические поля у насекомых, птиц и мелких позвоночных довольно низки при воздействии внешних электрических полей до 5000 мВ/м и магнитных полей до 50 мГс. Напротив, электрические поля, создаваемые в лиственных насаждениях теми же ЭМП, могут быть значительными. Расчеты, основанные на этих простых моделях, показывают, что поле, создаваемое вертикально ориентированным электрическим полем в 25-метровом дереве, может достигать 5000 мВ/м, а поле, создаваемое горизонтально ориентированным магнитным полем, — до 29 мВ/м.0,8 мВ/м. Сила приложенного или падающего электрического поля будет уменьшаться по мере удаления от провода антенны и из-за экранирования. Однако напряженность магнитного поля будет ослабевать вдали от антенного провода только на расстоянии. Поэтому на больших расстояниях от антенны поле, индуцированное в древостоях горизонтальным магнитным полем, может стать доминирующим фактором в результирующей дозе. (Более подробную информацию см. в Приложении B.)

Исследователей, участвующих в программе экологического мониторинга, не просили оценить дозы, полученные биотой от передающих антенн КНЧ ВМФ. Только исследователи, занимавшиеся изучением горной флоры, пытались это сделать. Поскольку по дозиметрии не было достаточной информации, комитет решил, что невозможно экстраполировать результаты программы мониторинга на другие ситуации, которые могут быть сопоставимы с условиями облучения.

Различия в действии между немодулированными сигналами 60 Гц и модулированными сигналами 76 Гц

Как упоминалось ранее, электрические и магнитные поля СНЧ, генерируемые антеннами системы связи, модулируются по частоте между 72 и 80 Гц (с преобладающей частотой 76 Гц). ), в отличие от ЭМП линий электропередач, которые не модулируются на частоте 60 Гц. К сожалению, мало информации о различиях между эффектами модулированных и немодулированных частот. Большинство исследований, предпринятых за последние 25 лет для понимания биологических эффектов низкочастотных ЭМП, были сосредоточены на воздействии немодулированных ЭМП на частотах сети 50-60 Гц (см., например, Anderson 19).90; ОРАУ 1992 г.; Тенфорд 1996; ОТА 1989 г.; NRC 1997). Было проведено мало исследований влияния модулированных сигналов частотой 76 Гц, создаваемых системой связи ELF.

Выводы относительно измерений ЭМП

IITRI проделал хорошую общую работу по характеристике электрических и магнитных полей сверхнизких частот вблизи мест обработки и контроля. В тех случаях, когда становилось очевидным, что требуется дополнительная информация, IITRI реагировал и проводил дополнительные измерения. Конкретные выводы таковы:

  • Несмотря на то, что были некоторые незначительные вопросы по конструкции прибора, оказалось, что связанные с этим ошибки были небольшими и не привели к изменению выводов IITRI относительно данных измерений.

  • Хорошо охарактеризованы пространственные и временные вариации магнитных полей над землей вблизи участков обработки и контроля.

  • Хорошо охарактеризованы пространственные и временные вариации электрических полей над землей на открытых площадках вблизи очистных и контрольных участков.

    В защищенных зонах, таких как деревья, необходимы более обширные измерения. По запросу их предоставили.

  • Электрические поля в земле зависят от локальной проводимости земли, поэтому для их характеристики требуются более тщательные измерения. По запросу IITRI предоставил инженерную поддержку для этих измерений.

  • Электрические поля Земли исследованы в свете ежегодных изменений электрических характеристик Земли. Большинство вариаций были скромными, но в этих полях происходили ежедневные и годовые изменения.

Сноски

1

“Результат” определяется следующим образом. Среднеквадратичные (среднеквадратические) величины трех прямоугольных составляющих поля определяются либо измерением, либо вычислением. синусоидально во времени, среднеквадратичное значение каждой составляющей представляет собой величину от нуля до пика, деленную на квадратный корень из 2. Результатом является квадратный корень из суммы квадратов этих трех среднеквадратичных значений

2

Сигнал 60 Гц линии электропередач на 30 дБ ниже сигнала передатчика 76 Гц; а самая сильная гармоника частоты линии электропередач (300 Гц) не менее чем на 60 дБ ниже сигнала передатчика. Спектры показывают гармоники частоты окружающего электроснабжения до 17-й гармоники и частоты передатчика до 11-й гармоники. Гармоники передатчика ниже основной на 35 дБ при 216 Гц и 240 Гц, на 50 дБ при 144 Гц и 160 Гц и на 55 дБ при 360 Гц и 400 Гц.

Измерения ЭМП, критерии воздействия и дозиметрия — оценка программы экологического мониторинга системы сверхнизкочастотной связи ВМС США

Изучение воздействия электрических и магнитных полей (ЭМП) на организмы включает точную оценку воздействия этих полей и того, что организм получает в результате воздействия. Экспозиция – это мера напряженности поля электрического или магнитного поля непосредственно снаружи организма за определенный период. Доза является мерой напряженности поля , индуцированного внутри организма в течение определенного периода времени. В первом разделе этой главы описывается, как IITRI охарактеризовал ЭМП вблизи передающих устройств. В последующих разделах обсуждаются проблемы, связанные с оценкой доз и учетом возможных эффектов, связанных с модуляцией сигнала.

Термин «ЭМП» применяется к переменному полю, создаваемому движущимися заряженными частицами. ЭМП характеризуются длиной волны (выраженной в метрах) и частотой (выраженной в герцах). Длина волны поля, умноженная на его частоту, равна скорость распространения. Полный диапазон частот ЭМП описывается как электромагнитный спектр. Обозначение «чрезвычайно низкочастотный» (ELF) обычно зарезервировано для частот в диапазоне от 3 Гц до 300 Гц. Большинство оборудования, используемого для генерации , передача и распределение электроэнергии в Соединенных Штатах генерирует ЭМП с частотой 60 Гц. Система связи ELF ВМФ использует принцип частотной модуляции, называемый минимальной манипуляцией. В этом типе модуляции частота сдвигается между 72 Гц и 80 Гц (с центром 76 Гц) в зависимости от того, должен ли передаваться на подводную лодку код «единица» или «ноль» (Запотоский и др. 19).96). Интенсивность электрических полей выражается в вольтах на метр (В/м), а магнитных полей выражается в миллигауссах (мГс). Дополнительная информация предоставлена ​​NIOSH, NIEHS и DOE (1996).

Характеристика электрических и магнитных полей

Для характеристики электрических и магнитных полей вблизи объектов экологического мониторинга IITRI измерил пространственные и временные характеристики следующих полей:

  • Магнитное поле в воздухе и на земле, создаваемое электрический ток в антенне и клеммах заземления.

  • Электрическое поле в земле, представляющее собой сумму полей, индуцированных магнитным полем, и тока, протекающего от заглубленных заземляющих клемм.

  • Электрическое поле в воздухе, возникающее из-за разности электрических потенциалов между антеннами и землей или создаваемое как побочный продукт электрического поля в земле.

  • Статическое геомагнитное поле Земли.

IITRI предоставил следующие размеры ЭМП вблизи передающих устройств (см. используемые приборы):

ТАБЛИЦА 2-1

Инструменты, используемые IITRI для измерения электромагнитных полей.

1.

Окружающая среда 60 Гц результирующая 1 ЭМП над землей.

2.

Немодулированные результирующие ЭМП 76 Гц над землей.

3.

Модулированные результирующие ЭМП 76 Гц над землей.

4.

Среднеквадратические значения гармоник ЭМП 60 Гц и 76 Гц над землей.

5.

Разность потенциалов земли в двух ортогональных направлениях и, на основании этого, результирующее электрическое поле в земле.

Возможность измерения магнитных полей низкого уровня зависит, среди прочего, от чувствительности прибора, используемого для анализа напряжения зонда магнитного поля. Согласно IITRI, оборудование для измерения магнитного поля было откалибровано путем измерения выходного напряжения зонда магнитного поля, помещенного в магнитное поле силой 100 мГс (Haradem et al. 19).94). В нем утверждается, что эта калибровка действительна при меньших уровнях поля на основании того факта, что датчик, построенный исключительно из пассивных компонентов, как известно, имеет выходной сигнал, линейный по отношению к напряженности поля. Наименьшая полномасштабная чувствительность прибора IITRI (Hewlett-Packard 3581A) составляла 0,1 мкВ, что соответствует уровню магнитного поля около 0,2 мГс. Ожидается, что измерения, намного более низкие, чем это (около 0,02 мГ или ниже), не будут очень точными. К счастью, наиболее важные сообщаемые уровни, использованные при создании участков обработки и контроля, а также при анализе экологических данных, были намного выше этих уровней и, как ожидается, будут точными представлениями магнитных полей. Однако ожидается, что сообщаемые уровни поля, такие как 0,0002 мГс, не будут точными.

Чтобы исключить возможность загрязнения экологических исследований гармониками или взаимодействиями между частотами линии электропередач и частотами антенны СНЧ, был измерен спектр зависимости напряженности поля от частоты. Все нежелательные сигналы оказались как минимум на 30 дБ ниже уровня частот антенны СНЧ, и поэтому было сочтено, что они не мешают экологическим исследованиям. Сообщалось, что среднеквадратические значения гармоник электрических и магнитных полей частотой 60 Гц и 76 Гц над землей либо ниже уровней обнаружения, либо «настолько низки, что их нельзя считать помехой». Спектры, измеренные IITRI на антенных терминалах с выключенным передатчиком и с включенным передатчиком, дают данные об окружающих полях 60 Гц и полях 76 Гц (J.R. Gauger, IITRI, письмо в Управление проекта систем связи ВМС США, 23 декабря 1985). Хотя приведенные спектральные данные относятся к наблюдениям за один день (11 декабря 1985 г. ), они подтверждают приведенное выше утверждение. 2 Признаки естественных резонансов Шумана в земной атмосфере и измеренное поведение гармоник также подтверждают качество наблюдений. Несколько явных ошибок в метках на спектрах согласуются с проблемами натурных наблюдений и легко устраняются. Однако таких ошибок было относительно мало.

Критерии воздействия для выбора места

Любой источник электрических и магнитных полей (например, передающие устройства сверхнизкой частоты и антенны) создает поля практически везде. По мере удаления от источника напряженность поля становится ниже либо из-за расстояния, либо из-за ослабления из-за препятствий (в случае электрических полей). Однако тогда человек будет двигаться к другим источникам, и генерируемые ими ЭМП будут увеличиваться по интенсивности по мере приближения. Поэтому невозможно выбрать место управления, где нет воздействия ЭМП СНЧ, генерируемого передающими средствами и антеннами ВМФ. Можно только выбрать места, которые имеют разные уровни воздействия ЭМП, генерируемого антеннами, и других источников, таких как линии электропередач.

IITRI помог исследователям выбрать участки для программы экологического мониторинга, определив, относятся ли они к категории обработки или контроля. Конкретные критерии, используемые для определения того, был ли участок лечебным или контрольным, были следующими:

, где T(76 Гц) – воздействие на лечебный участок из-за системы связи ELF, T(60 Гц) – облучение на лечебном участке из-за от линий электропередач, C(76 Гц) — воздействие на контрольную точку из-за системы связи ELF, а C(60 Гц) — воздействие на контрольную точку из-за линий электропередач.

Другими словами, интенсивность ЭМП 76 Гц в месте лечения должна была быть в 10 раз больше, чем интенсивность ЭМП 76 Гц в контрольной зоне. Кроме того, как на лечебных, так и на контрольных участках интенсивность ЭМП 76 Гц от антенн должна была в 10 раз превышать интенсивность ЭМП 60 Гц от близлежащих линий электропередач. Наконец, отношение интенсивностей полей частотой 60 Гц в пункте обработки и контроля должно быть в пределах от 0,1 до 10. Эти критерии применялись к ЭМП в воздухе и на земле при работе на полной мощности соответствующей передающей антенны. . Однако не было априорных доказательств того, что уменьшение воздействия на одну десятую должно привести к уменьшению на одну десятую (или меньше) эффекта, который можно было бы наблюдать при полном воздействии.

Изменение интенсивности поля в зависимости от расстояния означает, что каждое место подвергается воздействию пространственного градиента интенсивности, а не равномерной интенсивности по всему участку. Поэтому участки были классифицированы в соответствии с ежегодными измерениями, проводимыми в одной и той же точке каждый год.

Чтобы изолировать эффекты ЭМП КНЧ, парные участки обработки и контроля должны были быть как можно более похожими в отношении экологических переменных, включая почвы, листву, обилие видов и температуру, в зависимости от направленности исследования. Например, для исследования водно-болотных угодий в качестве участков обработки и контроля требовались аналогичные болота, тогда как для исследования насекомых-опылителей требовались участки с одинаковым обилием цветов. Несколько исследовательских групп столкнулись с трудностями при определении пар участков, отвечающих как критериям воздействия, так и экологическим критериям, как описано в главе 39.0007

Данные о воздействии, предоставленные исследователям

IITRI предоставил исследователям данные о магнитных полях и электрических полях в воздухе, а также электрических полях в земле. IITRI предоставил группам экологического мониторинга обширные данные о конкретных измерениях ЭМП КНЧ на каждом участке. Цель этих измерений состояла в том, чтобы позволить группам мониторинга определить индикаторы воздействия для различных частей участков обработки и попытаться связать индикаторы с соответствующими показателями экологического воздействия.

IITRI также предоставил данные о времени включения и выключения передатчика группам экологического мониторинга. Эти данные могут быть использованы для определения того, действительно ли место, охарактеризованное как место лечения, подвергалось воздействию ЭМП СНЧ от антенны в какое-либо конкретное время. Это важно, потому что передатчик не был включен постоянно, а место лечения подвергается воздействию только тогда, когда передатчик включен.

Использование формул для прогнозирования электрических и магнитных полей

Электрические и магнитные поля могут быть охарактеризованы либо физическими измерениями, либо теоретически, хотя оба метода сопряжены с трудностями. Физическое измерение напряженности поля может быть затруднено из-за ограничений измерительного оборудования и градиентов напряженности поля, вызванных расстоянием от источника и изменениями рельефа местности. Такие поля часто слишком сложны, чтобы их можно было адекватно охарактеризовать с помощью простых формул. Магнитное поле частотой 76 Гц было относительно устойчивым и могло быть хорошо охарактеризовано в пространстве с помощью простых формул. Однако для характеристики электрических полей были необходимы физические измерения. Пространственная зависимость электрических полей в земле не могла быть предсказана с помощью простых формул из-за пространственной изменчивости проводимости земли. Кроме того, имелись (обычно скромные) временные колебания из-за ежедневных и годовых изменений электропроводности Земли. Необходимо было провести тщательные исследования пространственных и временных изменений электрических полей Земли. Электрическое поле в воздухе имеет характеристики, которые находятся между характеристиками магнитного поля и электрического поля в земле. На открытой местности с ровным рельефом это поле хорошо характеризуется простыми формулами. Однако при наличии препятствий, таких как деревья, электрические поля в воздухе рассчитать сложно, и измерения дают только моментальные снимки, поскольку поле переменное. Например, ветер, движущийся сквозь листву, вызывает изменчивость электрических полей в воздухе.

IITRI предоставил несколько формул, которые можно использовать для расчета ЭМП сверхнизких частот вблизи антенн. Не все ограничения этих формул были указаны в отчетах, полученных комитетом. Авторы правильно указали, что формулы подходят только для точек поля на поверхности земли и вблизи антенн. Однако есть и другие ограничения. Во-первых, все формулы ограничены квазистатическим диапазоном частот, поэтому все соответствующие расстояния должны быть существенно меньше длины волны в земле. Во-вторых, каждый предполагает идеальную плоскую однородную землю. Некоторые из этих предположений верны, другие нет и исключают использование формул. Например, разумно рассчитать магнитные поля (либо в воздухе, либо в земле) с помощью простого закона Био-Савара, предоставленного IITRI, если известен ток антенны (какой он есть). Но обычно неверно предполагать, что проводимость земли однородна или не зависит от времени, для расчетов электрических полей в земле. Если необходимо учитывать изменения электропроводности в зависимости от глубины или горизонтального положения, формула, приведенная IITRI, недействительна. Фактически было обнаружено, что электропроводность существенно различалась на некоторых исследуемых участках и зависела от условий окружающей среды и, следовательно, от времени года. Вот почему следует быть очень осторожным в использовании этой формулы и почему необходимы дополнительные измерения электрических полей в земле. Осторожность, требуемая при использовании формулы для электрических полей в воздухе, находится между этими крайностями. Для однородной ровной местности без препятствий приведенные формулы удовлетворительны (при условии, что напряжение антенны известно). Однако в роще поля сильно искажены, и формула, предложенная IITRI, бесполезна.

Из формул, предоставленных IITRI, только формула магнитного поля широко использовалась исследователями. Эта формула использовалась только при попытке интерполировать магнитные поля в точках в пределах исследуемых участков. Для изучения горной флоры были проведены измерения магнитного поля частотой 76 Гц в нескольких точках вблизи висконсинской антенны. сравнивает измеренные с расчетными полями для высоты антенны 13,7 м и тока антенны 150 А. Измеренные и расчетные значения согласуются достаточно хорошо.

ТАБЛИЦА 2-2

Сравнение измеренных и рассчитанных магнитных полей.

К сожалению, IITRI не сообщил об измерениях напряжения между антенной и землей, которые можно было бы использовать для подтверждения результатов измерений электрических полей в воздухе. Однако это напряжение можно оценить следующим образом. Согласно Диллу (1984), одним из критериев проектирования клемм заземления было достижение максимального общего сопротивления заземления 6 Ом для обоих заземлений. На основании этого числа и предположения, что сопротивление антенны намного меньше сопротивления земли, напряжение между антенной и землей при полном токе (150 А) составляет 900 В. Согласно формуле, предоставленной IITRI, электрическое поле в воздухе непосредственно под антенной будет около 25 В/м. Измеренное электрическое поле под антенной для исследований мелких млекопитающих и гнездящихся птиц составляло 10-40 В/м. Из этого результата и теоретических оценок можно сделать вывод, что измерения находятся в разумном согласии.

Дозиметрия

Как указано выше, величины ЭМП, относящиеся к взаимодействию КНЧ с биологическими системами, представляют собой воздействие (напряженность поля сразу вне организма за период времени) и дозу (индуцированное поле внутри организмов за период времени). Последние величины могут быть выражены через напряженность индуцированного электрического поля, напряженность магнитного поля и индуцированный ток или плотность тока. Дозиметрия включает оценку величины и распределения индуцированных полей и токов в биологических организмах, подвергающихся воздействию ЭМП КНЧ. Индуцированные поля и токи не только являются функциями наложенных извне ЭМП, но и определяются свойствами ЭМП и геометрией подвергаемого воздействию организма и любых близлежащих объектов. Не следует ожидать, что такие индуцированные поля и токи на КНЧ будут составлять дозу.

Дозиметрические измерения индуцированных полей и токов не входили в план исследования. Тем не менее, чтобы получить некоторые данные об относительной силе индуцированных электрических полей в различных биотах, подвергшихся воздействию ЭМП частотой 76 Гц, комитет провел ряд анализов с использованием простых моделей, которые служат в качестве показателя индуцированных полей внутри организма. Среда воздействия КНЧ-ЭМП была охарактеризована на контрольных и лечебных участках посредством периодических обследований. Эта среда включала поля с частотой 76 Гц, создаваемые системой связи ELF, поля с частотой 60 Гц от линий электропередач и магнитное поле Земли. Поскольку длина волны 76 Гц намного длиннее самого длинного измерения организма, квазистатическая теория поля может быть надлежащим образом применена для расчета индуцированного электрического поля внутри тела организма (Майклсон и Лин 19).87).

Вкратце, результаты расчетов показывают, что индуцированные электрические поля у насекомых, птиц и мелких позвоночных довольно низки при воздействии внешних электрических полей до 5000 мВ/м и магнитных полей до 50 мГс. Напротив, электрические поля, создаваемые в лиственных насаждениях теми же ЭМП, могут быть значительными. Расчеты, основанные на этих простых моделях, показывают, что поле, создаваемое вертикально ориентированным электрическим полем в 25-метровом дереве, может достигать 5000 мВ/м, а поле, создаваемое горизонтально ориентированным магнитным полем, — до 29 мВ/м.0,8 мВ/м. Сила приложенного или падающего электрического поля будет уменьшаться по мере удаления от провода антенны и из-за экранирования. Однако напряженность магнитного поля будет ослабевать вдали от антенного провода только на расстоянии. Поэтому на больших расстояниях от антенны поле, индуцированное в древостоях горизонтальным магнитным полем, может стать доминирующим фактором в результирующей дозе. (Более подробную информацию см. в Приложении B.)

Исследователей, участвующих в программе экологического мониторинга, не просили оценить дозы, полученные биотой от передающих антенн КНЧ ВМФ. Только исследователи, занимавшиеся изучением горной флоры, пытались это сделать. Поскольку по дозиметрии не было достаточной информации, комитет решил, что невозможно экстраполировать результаты программы мониторинга на другие ситуации, которые могут быть сопоставимы с условиями облучения.

Различия в действии между немодулированными сигналами 60 Гц и модулированными сигналами 76 Гц

Как упоминалось ранее, электрические и магнитные поля СНЧ, генерируемые антеннами системы связи, модулируются по частоте между 72 и 80 Гц (с преобладающей частотой 76 Гц). ), в отличие от ЭМП линий электропередач, которые не модулируются на частоте 60 Гц. К сожалению, мало информации о различиях между эффектами модулированных и немодулированных частот. Большинство исследований, предпринятых за последние 25 лет для понимания биологических эффектов низкочастотных ЭМП, были сосредоточены на воздействии немодулированных ЭМП на частотах сети 50-60 Гц (см., например, Anderson 19).90; ОРАУ 1992 г.; Тенфорд 1996; ОТА 1989 г.; NRC 1997). Было проведено мало исследований влияния модулированных сигналов частотой 76 Гц, создаваемых системой связи ELF.

Выводы относительно измерений ЭМП

IITRI проделал хорошую общую работу по характеристике электрических и магнитных полей сверхнизких частот вблизи мест обработки и контроля. В тех случаях, когда становилось очевидным, что требуется дополнительная информация, IITRI реагировал и проводил дополнительные измерения. Конкретные выводы таковы:

  • Несмотря на то, что были некоторые незначительные вопросы по конструкции прибора, оказалось, что связанные с этим ошибки были небольшими и не привели к изменению выводов IITRI относительно данных измерений.

  • Хорошо охарактеризованы пространственные и временные вариации магнитных полей над землей вблизи участков обработки и контроля.

  • Хорошо охарактеризованы пространственные и временные вариации электрических полей над землей на открытых площадках вблизи очистных и контрольных участков.

    В защищенных зонах, таких как деревья, необходимы более обширные измерения. По запросу их предоставили.

  • Электрические поля в земле зависят от локальной проводимости земли, поэтому для их характеристики требуются более тщательные измерения. По запросу IITRI предоставил инженерную поддержку для этих измерений.

  • Электрические поля Земли исследованы в свете ежегодных изменений электрических характеристик Земли. Большинство вариаций были скромными, но в этих полях происходили ежедневные и годовые изменения.

Сноски

1

“Результат” определяется следующим образом. Среднеквадратичные (среднеквадратические) величины трех прямоугольных составляющих поля определяются либо измерением, либо вычислением. синусоидально во времени, среднеквадратичное значение каждой составляющей представляет собой величину от нуля до пика, деленную на квадратный корень из 2. Результатом является квадратный корень из суммы квадратов этих трех среднеквадратичных значений

2

Сигнал 60 Гц линии электропередач на 30 дБ ниже сигнала передатчика 76 Гц; а самая сильная гармоника частоты линии электропередач (300 Гц) не менее чем на 60 дБ ниже сигнала передатчика. Спектры показывают гармоники частоты окружающего электроснабжения до 17-й гармоники и частоты передатчика до 11-й гармоники. Гармоники передатчика ниже основной на 35 дБ при 216 Гц и 240 Гц, на 50 дБ при 144 Гц и 160 Гц и на 55 дБ при 360 Гц и 400 Гц.

Измерения ЭМП, критерии воздействия и дозиметрия — оценка программы экологического мониторинга системы сверхнизкочастотной связи ВМС США

Изучение воздействия электрических и магнитных полей (ЭМП) на организмы включает точную оценку воздействия этих полей и того, что организм получает в результате воздействия. Экспозиция – это мера напряженности поля электрического или магнитного поля непосредственно снаружи организма за определенный период. Доза является мерой напряженности поля , индуцированного внутри организма в течение определенного периода времени. В первом разделе этой главы описывается, как IITRI охарактеризовал ЭМП вблизи передающих устройств. В последующих разделах обсуждаются проблемы, связанные с оценкой доз и учетом возможных эффектов, связанных с модуляцией сигнала.

Термин «ЭМП» применяется к переменному полю, создаваемому движущимися заряженными частицами. ЭМП характеризуются длиной волны (выраженной в метрах) и частотой (выраженной в герцах). Длина волны поля, умноженная на его частоту, равна скорость распространения. Полный диапазон частот ЭМП описывается как электромагнитный спектр. Обозначение «чрезвычайно низкочастотный» (ELF) обычно зарезервировано для частот в диапазоне от 3 Гц до 300 Гц. Большинство оборудования, используемого для генерации , передача и распределение электроэнергии в Соединенных Штатах генерирует ЭМП с частотой 60 Гц. Система связи ELF ВМФ использует принцип частотной модуляции, называемый минимальной манипуляцией. В этом типе модуляции частота сдвигается между 72 Гц и 80 Гц (с центром 76 Гц) в зависимости от того, должен ли передаваться на подводную лодку код «единица» или «ноль» (Запотоский и др. 19).96). Интенсивность электрических полей выражается в вольтах на метр (В/м), а магнитных полей выражается в миллигауссах (мГс). Дополнительная информация предоставлена ​​NIOSH, NIEHS и DOE (1996).

Характеристика электрических и магнитных полей

Для характеристики электрических и магнитных полей вблизи объектов экологического мониторинга IITRI измерил пространственные и временные характеристики следующих полей:

  • Магнитное поле в воздухе и на земле, создаваемое электрический ток в антенне и клеммах заземления.

  • Электрическое поле в земле, представляющее собой сумму полей, индуцированных магнитным полем, и тока, протекающего от заглубленных заземляющих клемм.

  • Электрическое поле в воздухе, возникающее из-за разности электрических потенциалов между антеннами и землей или создаваемое как побочный продукт электрического поля в земле.

  • Статическое геомагнитное поле Земли.

IITRI предоставил следующие размеры ЭМП вблизи передающих устройств (см. используемые приборы):

ТАБЛИЦА 2-1

Инструменты, используемые IITRI для измерения электромагнитных полей.

1.

Окружающая среда 60 Гц результирующая 1 ЭМП над землей.

2.

Немодулированные результирующие ЭМП 76 Гц над землей.

3.

Модулированные результирующие ЭМП 76 Гц над землей.

4.

Среднеквадратические значения гармоник ЭМП 60 Гц и 76 Гц над землей.

5.

Разность потенциалов земли в двух ортогональных направлениях и, на основании этого, результирующее электрическое поле в земле.

Возможность измерения магнитных полей низкого уровня зависит, среди прочего, от чувствительности прибора, используемого для анализа напряжения зонда магнитного поля. Согласно IITRI, оборудование для измерения магнитного поля было откалибровано путем измерения выходного напряжения зонда магнитного поля, помещенного в магнитное поле силой 100 мГс (Haradem et al. 19).94). В нем утверждается, что эта калибровка действительна при меньших уровнях поля на основании того факта, что датчик, построенный исключительно из пассивных компонентов, как известно, имеет выходной сигнал, линейный по отношению к напряженности поля. Наименьшая полномасштабная чувствительность прибора IITRI (Hewlett-Packard 3581A) составляла 0,1 мкВ, что соответствует уровню магнитного поля около 0,2 мГс. Ожидается, что измерения, намного более низкие, чем это (около 0,02 мГ или ниже), не будут очень точными. К счастью, наиболее важные сообщаемые уровни, использованные при создании участков обработки и контроля, а также при анализе экологических данных, были намного выше этих уровней и, как ожидается, будут точными представлениями магнитных полей. Однако ожидается, что сообщаемые уровни поля, такие как 0,0002 мГс, не будут точными.

Чтобы исключить возможность загрязнения экологических исследований гармониками или взаимодействиями между частотами линии электропередач и частотами антенны СНЧ, был измерен спектр зависимости напряженности поля от частоты. Все нежелательные сигналы оказались как минимум на 30 дБ ниже уровня частот антенны СНЧ, и поэтому было сочтено, что они не мешают экологическим исследованиям. Сообщалось, что среднеквадратические значения гармоник электрических и магнитных полей частотой 60 Гц и 76 Гц над землей либо ниже уровней обнаружения, либо «настолько низки, что их нельзя считать помехой». Спектры, измеренные IITRI на антенных терминалах с выключенным передатчиком и с включенным передатчиком, дают данные об окружающих полях 60 Гц и полях 76 Гц (J.R. Gauger, IITRI, письмо в Управление проекта систем связи ВМС США, 23 декабря 1985). Хотя приведенные спектральные данные относятся к наблюдениям за один день (11 декабря 1985 г. ), они подтверждают приведенное выше утверждение. 2 Признаки естественных резонансов Шумана в земной атмосфере и измеренное поведение гармоник также подтверждают качество наблюдений. Несколько явных ошибок в метках на спектрах согласуются с проблемами натурных наблюдений и легко устраняются. Однако таких ошибок было относительно мало.

Критерии воздействия для выбора места

Любой источник электрических и магнитных полей (например, передающие устройства сверхнизкой частоты и антенны) создает поля практически везде. По мере удаления от источника напряженность поля становится ниже либо из-за расстояния, либо из-за ослабления из-за препятствий (в случае электрических полей). Однако тогда человек будет двигаться к другим источникам, и генерируемые ими ЭМП будут увеличиваться по интенсивности по мере приближения. Поэтому невозможно выбрать место управления, где нет воздействия ЭМП СНЧ, генерируемого передающими средствами и антеннами ВМФ. Можно только выбрать места, которые имеют разные уровни воздействия ЭМП, генерируемого антеннами, и других источников, таких как линии электропередач.

IITRI помог исследователям выбрать участки для программы экологического мониторинга, определив, относятся ли они к категории обработки или контроля. Конкретные критерии, используемые для определения того, был ли участок лечебным или контрольным, были следующими:

, где T(76 Гц) – воздействие на лечебный участок из-за системы связи ELF, T(60 Гц) – облучение на лечебном участке из-за от линий электропередач, C(76 Гц) — воздействие на контрольную точку из-за системы связи ELF, а C(60 Гц) — воздействие на контрольную точку из-за линий электропередач.

Другими словами, интенсивность ЭМП 76 Гц в месте лечения должна была быть в 10 раз больше, чем интенсивность ЭМП 76 Гц в контрольной зоне. Кроме того, как на лечебных, так и на контрольных участках интенсивность ЭМП 76 Гц от антенн должна была в 10 раз превышать интенсивность ЭМП 60 Гц от близлежащих линий электропередач. Наконец, отношение интенсивностей полей частотой 60 Гц в пункте обработки и контроля должно быть в пределах от 0,1 до 10. Эти критерии применялись к ЭМП в воздухе и на земле при работе на полной мощности соответствующей передающей антенны. . Однако не было априорных доказательств того, что уменьшение воздействия на одну десятую должно привести к уменьшению на одну десятую (или меньше) эффекта, который можно было бы наблюдать при полном воздействии.

Изменение интенсивности поля в зависимости от расстояния означает, что каждое место подвергается воздействию пространственного градиента интенсивности, а не равномерной интенсивности по всему участку. Поэтому участки были классифицированы в соответствии с ежегодными измерениями, проводимыми в одной и той же точке каждый год.

Чтобы изолировать эффекты ЭМП КНЧ, парные участки обработки и контроля должны были быть как можно более похожими в отношении экологических переменных, включая почвы, листву, обилие видов и температуру, в зависимости от направленности исследования. Например, для исследования водно-болотных угодий в качестве участков обработки и контроля требовались аналогичные болота, тогда как для исследования насекомых-опылителей требовались участки с одинаковым обилием цветов. Несколько исследовательских групп столкнулись с трудностями при определении пар участков, отвечающих как критериям воздействия, так и экологическим критериям, как описано в главе 39.0007

Данные о воздействии, предоставленные исследователям

IITRI предоставил исследователям данные о магнитных полях и электрических полях в воздухе, а также электрических полях в земле. IITRI предоставил группам экологического мониторинга обширные данные о конкретных измерениях ЭМП КНЧ на каждом участке. Цель этих измерений состояла в том, чтобы позволить группам мониторинга определить индикаторы воздействия для различных частей участков обработки и попытаться связать индикаторы с соответствующими показателями экологического воздействия.

IITRI также предоставил данные о времени включения и выключения передатчика группам экологического мониторинга. Эти данные могут быть использованы для определения того, действительно ли место, охарактеризованное как место лечения, подвергалось воздействию ЭМП СНЧ от антенны в какое-либо конкретное время. Это важно, потому что передатчик не был включен постоянно, а место лечения подвергается воздействию только тогда, когда передатчик включен.

Использование формул для прогнозирования электрических и магнитных полей

Электрические и магнитные поля могут быть охарактеризованы либо физическими измерениями, либо теоретически, хотя оба метода сопряжены с трудностями. Физическое измерение напряженности поля может быть затруднено из-за ограничений измерительного оборудования и градиентов напряженности поля, вызванных расстоянием от источника и изменениями рельефа местности. Такие поля часто слишком сложны, чтобы их можно было адекватно охарактеризовать с помощью простых формул. Магнитное поле частотой 76 Гц было относительно устойчивым и могло быть хорошо охарактеризовано в пространстве с помощью простых формул. Однако для характеристики электрических полей были необходимы физические измерения. Пространственная зависимость электрических полей в земле не могла быть предсказана с помощью простых формул из-за пространственной изменчивости проводимости земли. Кроме того, имелись (обычно скромные) временные колебания из-за ежедневных и годовых изменений электропроводности Земли. Необходимо было провести тщательные исследования пространственных и временных изменений электрических полей Земли. Электрическое поле в воздухе имеет характеристики, которые находятся между характеристиками магнитного поля и электрического поля в земле. На открытой местности с ровным рельефом это поле хорошо характеризуется простыми формулами. Однако при наличии препятствий, таких как деревья, электрические поля в воздухе рассчитать сложно, и измерения дают только моментальные снимки, поскольку поле переменное. Например, ветер, движущийся сквозь листву, вызывает изменчивость электрических полей в воздухе.

IITRI предоставил несколько формул, которые можно использовать для расчета ЭМП сверхнизких частот вблизи антенн. Не все ограничения этих формул были указаны в отчетах, полученных комитетом. Авторы правильно указали, что формулы подходят только для точек поля на поверхности земли и вблизи антенн. Однако есть и другие ограничения. Во-первых, все формулы ограничены квазистатическим диапазоном частот, поэтому все соответствующие расстояния должны быть существенно меньше длины волны в земле. Во-вторых, каждый предполагает идеальную плоскую однородную землю. Некоторые из этих предположений верны, другие нет и исключают использование формул. Например, разумно рассчитать магнитные поля (либо в воздухе, либо в земле) с помощью простого закона Био-Савара, предоставленного IITRI, если известен ток антенны (какой он есть). Но обычно неверно предполагать, что проводимость земли однородна или не зависит от времени, для расчетов электрических полей в земле. Если необходимо учитывать изменения электропроводности в зависимости от глубины или горизонтального положения, формула, приведенная IITRI, недействительна. Фактически было обнаружено, что электропроводность существенно различалась на некоторых исследуемых участках и зависела от условий окружающей среды и, следовательно, от времени года. Вот почему следует быть очень осторожным в использовании этой формулы и почему необходимы дополнительные измерения электрических полей в земле. Осторожность, требуемая при использовании формулы для электрических полей в воздухе, находится между этими крайностями. Для однородной ровной местности без препятствий приведенные формулы удовлетворительны (при условии, что напряжение антенны известно). Однако в роще поля сильно искажены, и формула, предложенная IITRI, бесполезна.

Из формул, предоставленных IITRI, только формула магнитного поля широко использовалась исследователями. Эта формула использовалась только при попытке интерполировать магнитные поля в точках в пределах исследуемых участков. Для изучения горной флоры были проведены измерения магнитного поля частотой 76 Гц в нескольких точках вблизи висконсинской антенны. сравнивает измеренные с расчетными полями для высоты антенны 13,7 м и тока антенны 150 А. Измеренные и расчетные значения согласуются достаточно хорошо.

ТАБЛИЦА 2-2

Сравнение измеренных и рассчитанных магнитных полей.

К сожалению, IITRI не сообщил об измерениях напряжения между антенной и землей, которые можно было бы использовать для подтверждения результатов измерений электрических полей в воздухе. Однако это напряжение можно оценить следующим образом. Согласно Диллу (1984), одним из критериев проектирования клемм заземления было достижение максимального общего сопротивления заземления 6 Ом для обоих заземлений. На основании этого числа и предположения, что сопротивление антенны намного меньше сопротивления земли, напряжение между антенной и землей при полном токе (150 А) составляет 900 В. Согласно формуле, предоставленной IITRI, электрическое поле в воздухе непосредственно под антенной будет около 25 В/м. Измеренное электрическое поле под антенной для исследований мелких млекопитающих и гнездящихся птиц составляло 10-40 В/м. Из этого результата и теоретических оценок можно сделать вывод, что измерения находятся в разумном согласии.

Дозиметрия

Как указано выше, величины ЭМП, относящиеся к взаимодействию КНЧ с биологическими системами, представляют собой воздействие (напряженность поля сразу вне организма за период времени) и дозу (индуцированное поле внутри организмов за период времени). Последние величины могут быть выражены через напряженность индуцированного электрического поля, напряженность магнитного поля и индуцированный ток или плотность тока. Дозиметрия включает оценку величины и распределения индуцированных полей и токов в биологических организмах, подвергающихся воздействию ЭМП КНЧ. Индуцированные поля и токи не только являются функциями наложенных извне ЭМП, но и определяются свойствами ЭМП и геометрией подвергаемого воздействию организма и любых близлежащих объектов. Не следует ожидать, что такие индуцированные поля и токи на КНЧ будут составлять дозу.

Дозиметрические измерения индуцированных полей и токов не входили в план исследования. Тем не менее, чтобы получить некоторые данные об относительной силе индуцированных электрических полей в различных биотах, подвергшихся воздействию ЭМП частотой 76 Гц, комитет провел ряд анализов с использованием простых моделей, которые служат в качестве показателя индуцированных полей внутри организма. Среда воздействия КНЧ-ЭМП была охарактеризована на контрольных и лечебных участках посредством периодических обследований. Эта среда включала поля с частотой 76 Гц, создаваемые системой связи ELF, поля с частотой 60 Гц от линий электропередач и магнитное поле Земли. Поскольку длина волны 76 Гц намного длиннее самого длинного измерения организма, квазистатическая теория поля может быть надлежащим образом применена для расчета индуцированного электрического поля внутри тела организма (Майклсон и Лин 19).87).

Вкратце, результаты расчетов показывают, что индуцированные электрические поля у насекомых, птиц и мелких позвоночных довольно низки при воздействии внешних электрических полей до 5000 мВ/м и магнитных полей до 50 мГс. Напротив, электрические поля, создаваемые в лиственных насаждениях теми же ЭМП, могут быть значительными. Расчеты, основанные на этих простых моделях, показывают, что поле, создаваемое вертикально ориентированным электрическим полем в 25-метровом дереве, может достигать 5000 мВ/м, а поле, создаваемое горизонтально ориентированным магнитным полем, — до 29 мВ/м.0,8 мВ/м. Сила приложенного или падающего электрического поля будет уменьшаться по мере удаления от провода антенны и из-за экранирования. Однако напряженность магнитного поля будет ослабевать вдали от антенного провода только на расстоянии. Поэтому на больших расстояниях от антенны поле, индуцированное в древостоях горизонтальным магнитным полем, может стать доминирующим фактором в результирующей дозе. (Более подробную информацию см. в Приложении B.)

Исследователей, участвующих в программе экологического мониторинга, не просили оценить дозы, полученные биотой от передающих антенн КНЧ ВМФ. Только исследователи, занимавшиеся изучением горной флоры, пытались это сделать. Поскольку по дозиметрии не было достаточной информации, комитет решил, что невозможно экстраполировать результаты программы мониторинга на другие ситуации, которые могут быть сопоставимы с условиями облучения.

Различия в действии между немодулированными сигналами 60 Гц и модулированными сигналами 76 Гц

Как упоминалось ранее, электрические и магнитные поля СНЧ, генерируемые антеннами системы связи, модулируются по частоте между 72 и 80 Гц (с преобладающей частотой 76 Гц). ), в отличие от ЭМП линий электропередач, которые не модулируются на частоте 60 Гц. К сожалению, мало информации о различиях между эффектами модулированных и немодулированных частот. Большинство исследований, предпринятых за последние 25 лет для понимания биологических эффектов низкочастотных ЭМП, были сосредоточены на воздействии немодулированных ЭМП на частотах сети 50-60 Гц (см., например, Anderson 19).90; ОРАУ 1992 г.; Тенфорд 1996; ОТА 1989 г.; NRC 1997). Было проведено мало исследований влияния модулированных сигналов частотой 76 Гц, создаваемых системой связи ELF.

Выводы относительно измерений ЭМП

IITRI проделал хорошую общую работу по характеристике электрических и магнитных полей сверхнизких частот вблизи мест обработки и контроля. В тех случаях, когда становилось очевидным, что требуется дополнительная информация, IITRI реагировал и проводил дополнительные измерения. Конкретные выводы таковы:

  • Несмотря на то, что были некоторые незначительные вопросы по конструкции прибора, оказалось, что связанные с этим ошибки были небольшими и не привели к изменению выводов IITRI относительно данных измерений.

  • Хорошо охарактеризованы пространственные и временные вариации магнитных полей над землей вблизи участков обработки и контроля.

  • Хорошо охарактеризованы пространственные и временные вариации электрических полей над землей на открытых площадках вблизи очистных и контрольных участков.

    В защищенных зонах, таких как деревья, необходимы более обширные измерения. По запросу их предоставили.

  • Электрические поля в земле зависят от локальной проводимости земли, поэтому для их характеристики требуются более тщательные измерения. По запросу IITRI предоставил инженерную поддержку для этих измерений.

  • Электрические поля Земли исследованы в свете ежегодных изменений электрических характеристик Земли. Большинство вариаций были скромными, но в этих полях происходили ежедневные и годовые изменения.

Сноски

1

“Результат” определяется следующим образом. Среднеквадратичные (среднеквадратические) величины трех прямоугольных составляющих поля определяются либо измерением, либо вычислением. синусоидально во времени, среднеквадратичное значение каждой составляющей представляет собой величину от нуля до пика, деленную на квадратный корень из 2. Результатом является квадратный корень из суммы квадратов этих трех среднеквадратичных значений

2

Сигнал 60 Гц линии электропередач на 30 дБ ниже сигнала передатчика 76 Гц; а самая сильная гармоника частоты линии электропередач (300 Гц) не менее чем на 60 дБ ниже сигнала передатчика. Спектры показывают гармоники частоты окружающего электроснабжения до 17-й гармоники и частоты передатчика до 11-й гармоники. Гармоники передатчика ниже основной на 35 дБ при 216 Гц и 240 Гц, на 50 дБ при 144 Гц и 160 Гц и на 55 дБ при 360 Гц и 400 Гц.

Измерение и расчет ЭДС

Измерение электрических и магнитных полей

 Это довольно технический отчет о принципах измерения — для более простой версии см. предыдущий переключатель.

Первые коммерческие приборы, разработанные специально для измерения полей промышленной частоты, стали доступны в 1980-х годах. В настоящее время доступно множество приборов, различающихся по различным характеристикам:

(a) Количество осей обнаружения . Нет датчиков, которые непосредственно оценивают результирующее поле в случайном направлении в пространстве; Датчики обычно измеряют поле в одном направлении. Счетчик может иметь один датчик. Если это выровнено пользователем с направлением максимального поля, это даст показание максимального поля в одном направлении; общее результирующее поле может быть в 1,0–1,41 раза больше этого значения в зависимости от степени поляризации. Если счетчик имеет три ортогональных датчика, результирующее поле может быть получено из трех значений, измеренных путем сложения суммы корней квадратов: Результирующее = (X 2 +Y 2 +Z 2 ) 1/2 .

Это результирующее значение не зависит от ориентации измерителя, что значительно упрощает использование измерителя.
Подробнее об эллиптически поляризованных полях
   
(b) Мера поля . Возможны различные меры синусоиды, например. пиковое значение, выпрямленное среднее значение, среднеквадратичное значение (среднеквадратичное значение). Для одной частоты, т. е. чистой синусоидальной волны, их можно масштабировать для получения одного и того же результата, но при наличии гармоник они могут значительно различаться. В отсутствие известного биофизического механизма нет убедительных оснований утверждать, что какая-то одна мера верна. Однако по аналогии с другими областями измерительной науки существует предположение, что среднеквадратичное значение является предпочтительной мерой. Некоторые измерители фиксируют реальную форму сигнала для последующего анализа.
   
(c) Частотная характеристика . Инструменты могут быть чувствительны к одной частоте, например. 50 Гц или 60 Гц или в диапазоне частот. При чувствительности к диапазону частот отклик может быть плоским или пропорциональным частоте. Плоская частотная характеристика от 20 или 30 Гц до нескольких килогерц обычно считается подходящей для многих измерений общего назначения.
   
(d) Размер датчиков . Датчики можно сделать маленькими — несколько миллиметров — и, следовательно, способными исследовать изменения поля на небольших расстояниях. Однако также могут быть случаи, когда желательно использовать датчики большего размера, которые измеряют среднее поле по своей площади. Вот два разных способа изготовления прибора для измерения магнитного поля:

     

Датчик слева имеет три катушки, центрированные друг относительно друга. Они с воздушным сердечником и для получения необходимой чувствительности имеют тысячи витков провода. Эти примеры имеют площадь 10 см.

Датчик справа имеет намного меньшие катушки, чтобы уменьшить общий измеритель. Для получения чувствительности, несмотря на меньшие размеры, катушки имеют железные сердечники. Это означает, что они не могут быть центрированы в одной и той же точке; они расположены отдельно, под прямым углом друг к другу (две плоские на плате внизу слева, третья, вертикальная, катушка снабжена белой механической опорой внизу справа).

    
(e) Считывание и регистрация . Счетчики могут иметь аналоговые или цифровые дисплеи. Они могут отображать значение только в режиме реального времени или могут регистрировать значения с различной степенью сложности и вычислять различные параметры поля, такие как средние или максимальные значения.
   
Учитывая различия в возможностях, предоставляемых счетчиком, неизбежны различия в размере, весе и потреблении батареи. Некоторые измерители наиболее подходят для детальных обследований экспертами; другие маленькие и достаточно легкие, чтобы добровольцы могли носить их в течение длительного времени.

Не существует «правильного» или «лучшего» счетчика. Лучший счетчик для использования зависит от цели, для которой он будет использоваться.

Измерение магнитных полей

Для измерения магнитных полей широко используются три различных датчика:
 
(a) Поисковые катушки . Простейшие измерители измеряют напряжение, индуцированное в катушке провода. Для синусоидально изменяющегося магнитного поля B с частотой f напряжение V, индуцируемое в катушке, определяется выражением:

V=-2 π f B 0 A cos(ω t)

, где ω = 2 π f — частота поля, A — площадь петли, B 0 — составляющая B, перпендикулярная петле.

Напряжение, индуцированное данным полем, увеличивается при добавлении дополнительных витков провода или ферромагнитного сердечника – см. примеры выше. Для предотвращения помех от электрических полей датчик магнитного поля должен быть экранирован. Если измеритель используется для обследований или измерений индивидуального облучения, частоты ниже приблизительно 30 Гц должны быть отфильтрованы, чтобы устранить напряжения, наведенные в зонде движением измерителя в магнитном поле Земли.
   
(b) Феррозондовые магнитометры . Они обнаруживают магнитное поле по асимметрии, которую оно создает в ферромагнитном материале, преднамеренно приводящемся в магнитное насыщение попеременно в противоположных направлениях на высокой частоте.
   
(c) Устройства на эффекте Холла . Датчик предназначен для измерения поперечного напряжения Холла на тонкой полоске полупроводникового материала, по которой течет продольный ток.

В большинстве практичных приборов для промышленных частот используются поисковые катушки, либо одна катушка, либо три ортогональных катушки. Сами катушки можно сделать как можно меньше, с ферромагнитным сердечником для повышения чувствительности, для использования в персональных экспонометрах, где размер и вес являются важными критериями; или они могут быть больше, часто 0,1 м в поперечнике, чтобы повысить чувствительность и обеспечить некоторое пространственное усреднение. Феррозондовые магнитометры нельзя сделать такими же маленькими или дешевыми, но они имеют то преимущество, что они реагируют как на постоянные поля, так и на переменные. Устройства Холла мало используются, поскольку их разрешение хуже и они страдают дрейфом, но их можно использовать в более высоких полях.

Измерение электрических полей

Измерители электрических полей обычно используют в качестве датчиков две параллельные проводящие пластины. Альтернативные датчики, т.е. основанные на вращении поляризованного света, встречаются реже.

Доступны трехосевые измерители электрического поля, но более распространены одноосевые измерители. Отчасти это связано с тем, что трехосевые измерители для электрических полей труднее изготовить, чем для магнитных полей, а отчасти потому, что в одной обычной ситуации измерения, вблизи земли под или рядом с воздушными линиями электропередач, электрическое поле линейно поляризовано и в известном направление (вертикальное), поэтому вполне достаточно одноосевого измерителя.

Человек, держащий измеритель электрического поля, может возмущать поле. Для измерения невозмущенного поля измеритель обычно подвешивают на конце длинного непроводящего горизонтального стержня или вертикального штатива. Показания считываются с расстояния на дисплее подходящего размера, записываются внутри счетчика для последующего анализа или передаются на считывающее устройство по оптоволоконному кабелю. Это может уменьшить возмущение до приемлемого уровня. Однако, учитывая легкость возмущения электрических полей, легко сделать ошибочные измерения, особенно при наличии:

  • экстремальные значения температуры и влажности;
  • недостаточное расстояние зонда от исследователя;
  • нестабильность положения счетчика;
  • потеря непроводящих свойств опорного стержня.

Электрические поля также можно измерять в фиксированных местах, например под линиями электропередачи или в лабораторных экспозиционных камерах путем измерения тока, собираемого плоской проводящей пластиной, расположенной на уровне земли. Для синусоидальных полей плотность электрического потока можно рассчитать по площади пластины (A), диэлектрической проницаемости вакуума, частоте (f) и измеренному току, индуцируемому в пластине, в выражении ниже:

E=I rms /2πfε 0 A

Персональные экспонометры существуют для электрических полей. Однако ношение измерительного прибора на теле приводит к непредсказуемым изменениям измеряемого электрического поля. Как правило, при измерении воздействия электрических полей на большие группы людей прибор помещают в нарукавную повязку, карман рубашки или поясную сумку. Возмущение окружающего поля телом не позволяет получить абсолютное значение поля, и в лучшем случае среднее значение таких измерений отражает относительный уровень облучения.

индукция – Измерение падения напряжения от индуцированного тока

спросил

Изменено 6 лет назад

Просмотрено 4к раз

$\begingroup$

У меня проблемы с соединением падения напряжения и наведенного тока. Представьте, что у вас есть треугольная петля, состоящая из трех резисторов. Вы помещаете эту петлю в постоянно меняющееся магнитное поле. Это изменяющееся магнитное поле вызовет ЭДС и результирующий индуцированный ток в петле.

Теперь представьте, что я приложил вольтметр к одному из резисторов. Что показывает вольтметр? $IR$ или $2IR$? Если посмотреть на ситуацию с одной стороны, вы измеряете падение напряжения на одном резисторе, поэтому $V$ будет равно $IR$. Но если вы посмотрите на это с другой стороны, вы также измеряете падение напряжения на другой половине цепи, которая содержит два резистора (отсюда падение напряжения составляет $2IR$).

Я пытался смоделировать эту ситуацию в своей школьной лаборатории физики и вижу, что на резисторе есть падение напряжения, поэтому ответ не “падение напряжения отсутствует”. Моя модель не была идеальной, но казалось, что вольтметр показывает $3IR$? Как это могло быть? 9b \mathbf{E} \cdot d\mathbf{s}$ по пути из $a$ в $b$. Обычно это значение не зависит от пути, поэтому можно говорить о падении напряжения между $a$ и $b$. В этой ситуации значение действительно зависит от пути — вы можете получить $IR$ или $2IR$ или любое другое значение в зависимости от того, как вы установили провода вольтметра. Например, если вы обмотаете выводы вольтметра вокруг треугольника несколько раз, вы можете получить $25IR$.

$\endgroup$

$\begingroup$

Я бы попробовал объяснить без привлечения математики!

Здесь мы говорим об индуцированном электрическом поле, так как работа, совершаемая по разным путям для такого поля, различна, нет определения разности потенциалов! Это прямо означает, что вы не можете измерить какую-либо разность потенциалов, которую надеялись измерить!

Даже не предполагается, что ЭДС должна быть одинаковой как для 1-го резистора, так и для 2-х резисторов; Все, что здесь предполагается, — это ЭДС индукции, которая генерирует ток в соответствии с законами Ленца.

Кроме того, я полагаю, вы думали, что ваш эксперимент дал результаты для $ I×3R $, потому что петля вольтметра начала бы циркулировать свой собственный ток, давая большее значение, чем то, что вы ожидали увидеть!

Я бы предложил следующий эксперимент, чтобы получить значение ЭДС индукции: 1. Поместите в цепь идеалистический амперметр, чтобы измерить ток в цепи. 2. Найдите чистое сопротивление цепи для 3 резисторов, объединенных, как вы сказали, это будет рассматриваться как параллельное из $R$ и $2R$. 3. Теперь вы можете использовать основной закон Ома, чтобы найти ЭДС индукции.

Поскольку для расчета тока в цепи мы используем обратную сторону этого метода, описанный выше метод должен дать удовлетворительные результаты!

$\endgroup$

$\begingroup$

То, что показывает вольтметр в данном случае, не является классическим падением напряжения , поскольку само понятие напряжения, как скалярной функции, градиент которой дает поле, становится избыточным. То, что он отображает косвенно, – это ток через него, который в случае ЭДС индуцирования не связан с «падением потенциала» на его конце. (в качестве показаний он отобразит $V$, где $V=Ir$, $I$ ток через него, а $r$ сопротивление вольтметра.

В качестве примера рассмотрим случай, указанный в вопросе. Три резистора сопротивлением $R=5$ каждый соединены треугольником. Вольтметр сопротивления $r=100$ подключен через две вершины. Пусть изменение магнитного потока ограничивается только треугольной частью и составляет $1 вебер/с$. Следовательно, линейный интеграл поля вдоль линии, полностью включающей область с изменяющимся магнитным потоком, должен быть равен $1 вольт$. Пусть ток через вольтметр равен $I_1$(от А к С), а через два резистора (кроме того, к которому присоединен вольтметр) равен $I$(по часовой стрелке). Ток через сопротивление, к которому подключен вольтметр, равен $I-I_1$(от А до С).

Во-первых, рассмотрим линейный интеграл вдоль треугольной части по часовой стрелке. Интеграл даст $$-2IR-(I-I_1)R=1$$ $$-15I+5I_1=1$$ Рассматривая линейный интеграл вдоль $C\rightarrow B\rightarrow A\rightarrow \text{Вольтметр}\rightarrow C$ по часовой стрелке, получим $$-2IR-I_1r=1$$, так как в обоих случаях изменение магнитного потока (скорость изменения во времени) одинаково, так как изменение ограничено треугольной областью, которая полностью ограничена обоими линейными интегралами.

Оставить комментарий